Answers to the Flood’s 50 hardest problematic questions.

There are a wide variety of supernatural events and miracles recorded in the Bible. But out of all of them, the one story or event that is most often criticized, scoffed at, or compromised is the story of Noah’s Arkand the Flood. For skeptics it is an easy event to criticize, but for Bible believing Christians it often creates a divide; those that believe it is 100% true and those that assume it is merely a fictional tale that is full of morals and analogies that will help followers of Christ better understand God. I admit that my own skepticism of the Bible held out the longest when it came to Noah and the Flood. After all, it just seems so impossible. And when scientists are asked about the subject, the answers are almost unanimous: Impossible or No Evidence!

What you will hopefully find though, as I found, is that those who think such events are impossible usually don’t know the actual Biblical story, the Creationist model, and have presuppositions about what the evidence in geology reveals. For example, geologists are taught a naturalistic uniformitarian synthesis in geology in their respective universities which explains all geological evidence as a result of slow and gradual natural causes over millions of years. Of course such a regimental syllabus would reveal no evidence of a flood because a global flood would be a catastrophic event of supernatural origin that cannot fit in a naturalistic uniformitarian framework. In other words, the geological evidence for the global flood is the same evidence for slow and gradual uniformitarian naturalism, there are just two opposite presuppositions influencing how that evidence is interpreted. But that will be discussed in more detail later on.

The Supernatural Element

The most important factor that is often denied is the supernatural element. We cannot overlook the supernatural aspect of Noah’sArkand the Flood because, let’s face it, it was an act of God. God is supernatural, His miracles supernatural. Since non-Christian skeptics presuppose there is no supernatural, only the natural, they will always reject the notion of two of all the animals of the world being crammed onto a wooden boat while the rest of the world is flooded, because such an event is not naturally possible, and the supernatural cannot be considered because there is no such thing. This is often an easy out for non-Christian skeptics. While on the other side of the coin, many Christians who believe in the supernatural chalk everything up to a miracle of God, and therefore possible, which is also an easy out, though I don’t disagree with that notion. None the less, this is not satisfying for the non-Christian skeptic.

Unfortunately for all you non-Christian skeptics reading this article there will be many aspects of the Biblical story of Noah’s Arkand the great flood that will not have any other explanation other than that of supernatural intervention from God. But this should be expected considering the whole event was initiated and orchestrated by God. Thus, why would anyone expect naturalistic causes for the entire event? Doing so would render God’s role in the event null and void, defeating the whole point of the event and its subsequent inclusion in the Book of Genesis for that matter. As geologist Dr. Andrew Snelling admits, “…it is simply not possible to have any kind of Genesis Flood without acknowledging the presence of supernatural elements. On the other hand, it is far from necessary to indulge in an ‘endless supply of miracles to make a universal Flood feasible.’”[1]

With that, I will attempt to provide a detailed explanation and defense for some of the most popular criticisms or “problems” of the story of Noah and the global flood. I will, for the most part, provide natural causes and available evidences when possible and required, but ultimately there will be many factors that will and can only be explained via the supernatural. My hope is that after reading this you will understand that it is perfectly rational and reasonable to adhere to this story being a true story.

PROBLEM OUTLINE:

Pre-Flood Problems; #1-6

Flood Problems; #7-35

Post-Flood Problems; #36-46

Theological/Philosophical Problems; #47-50

PRE-FLOOD

Problem 1: Building theArk.It is impossible for one man and his family to build an ark 450-510 ft long in ancient times.

Answer: Though it is said in the Bible that Noah and his four sons built the ark, the Bible does not say that Noah did not hire anyone else to help. There could have been a lot more than just five men building the ark. However, Noah and his three sons could have built the ark by themselves considering they had years to do it. In addition, it should not be neglected to point out that during Noah’s time humans were not so primitive, but instead very capable; forging metal, building cities, and crafting complex musical instruments.[2] Therefore, their tools, machines and technical capabilities were not primitive. Furthermore, if one considers a technology lost scenario, it can be assumed that Noah and his family had even greater capability than we know of. Technology lost, is a scenario that has happened frequently in human history where a technology is lost and forgotten and then re-invented long afterwards. For example, it was the Romans that first invented concrete, the technology was lost after the fall of the Roman Empire, and it was re-invented in the 1800s.[3] Such a situation would be very plausible in the event that a global flood killed every human on earth and subsequently the technologies they had, which would need to be reinvented later.

Problem 2: Wooden Ship. TheArkis too large to have been made from wood. The wood would buckle and surely not maintain hull integrity in rough seas.

Answer: The ark’s dimensions were built at a 6:1 ratio, which is very similar to modern ocean going vessels. Such dimensions are common in ocean vessels because they’re best for handling pitching, rolling and overall stability in rough seas.[4] The Danish apparently modeled barges called Fleuten after the ark (though not as large) which were incredibly seaworthy and almost impossible to capsize.[5] Many studies and experiments have been undertaken to test the seaworthiness of the Ark, which today is considered to have had hydrodynamically stable dimensions. As Hydraulic Engineer Dr. Henry Morris states, “… the ark as designed was highly stable, admirably suited for its purpose of riding out the storms of the year of the Great Flood.”[6] A more recent experiment conducted by the Research Institute of Ships and Engineering in Korea (KRISO) found that the Ark’s stability was thirteen times more stable than that of standard requirements regulated by the American Bureau of Shipping.[7] They also found that theArk’s dimensions were the most efficient for combined stability, strength and comfort. It should also be noted that Dr. Seon Hong, who headed up the study at KRISO, is an evolutionist, and not a biased creationist as some might claim.

The use of wood as a building material is not necessarily a problem either. Though the Bible doesn’t specifically mention the technical method by which the hull was constructed, many engineering techniques used in ancient ship building were actually sophisticated and complex. As mechanical engineer Tim Lovett writes, “Ancient shipbuilders usually began with a shell of planks (strakes) and then built internal framing (ribs) to fit inside… Some used overlapping (clinker) plates that were dowelled or nailed, while others used rope to sew the planks together. The ancient Greeks used a sophisticated system where the planks were interlocked with thousands of precise mortise and tenon joints. The resulting hull was strong enough to ram another ship, yet light enough to be hauled onto a beach by the crew. If this is what the Greeks could do centuries before Christ, what could Noah do centuries after Tubal-Cain invented forged metal tools?”[8] Lovett brings up the point that metal could have been used as well in the construction, since the Bible doesn’t necessarily say it wasn’t used. Lastly, it should not be overlooked that the structural integrity of wood on ships is often challenged when subjected to speed. TheArk however was only required to float and remain stable. Removing the stresses of speed greatly reduces the demands placed on the wooden hull of theArk. Thus when considering the dimensions (6:1 ratio) of the Ark, the capabilities of ship building in ancient times, the possible use of metal reinforcement, and the removal of speed stresses, it is clear that the wooden Ark, despite it’s large size could have endured the stresses of relentless violent seas.

Problem 3: Gathering the Animals. It is not feasible for every type of animal on the planet to travel to Noah. How did animals that can’t travel over land well, like sloths and penguins, do it? What about animals that can only survive in very precise climates? Or animals that require a very specific diet? What about land animals separated by thousands of miles of ocean?

Answer: The solution to this problem is one of world view. Skeptics place Noah’s Arkwithin the naturalist framework of the earth being billions of years old, and therefore, the continents already separated with vast oceans between them at the time and very distinct regions throughout. However, the creationist model proposes the continents split during the flood, and therefore, the continents had not been separated before the flood, but was instead all locked together in one super continent often referred to as Pangea. Thus, making it possible for all land animals to reach Noah by land.

Likewise, the specification of animals being suited strictly to their environment wasn’t as defined prior to the flood as it is today after the flood. Artic or very arid regions of earth may have not been present prior to the flood. Huge mountain ranges and impenetrable jungles may have not been present either. With one super landmass it is more likely that animals were more widely distributed then they are in the present where they are mostly isolated in particular regions. Furthermore, specialization in these animals and there ecological niches or limited diets may have occurred only after the flood when the post-flood regions were present. This is more plausible when one considers the genetic bottlenecking and dispersal patterns of flora and fauna that would surely have occurred in a global flood situation.

Therefore, the animals may have looked differently, eaten differently, and lived in a climate differently prior to the flood, than how they do now. The varying distinctions between animal species we see today could have very well developed among the animals after the flood. And without such distinct regions on earth prior to the flood, it allows the possibility for the animals to have been living very close to Noah or at least within traveling distance.

As far as how all the animals found Noah… That, unfortunately for skeptics, requires supernatural intervention. There is no natural process one could propose to explain how all the animals navigated their way to the ark accept to point out the migratory capabilities of birds and other animals that can travel very precise courses over thousands of miles. This ability may have played a role in the animals getting to the ark, but for the most part we have to assume God orchestrated this event.

Problem 4: Loading the Animals. There was not nearly enough time to load all the animals on the ark. The sheer number of animals loaded on the Ark would be in the millions, now calculate how long it would take to load each couple, two by two, and now we’re looking at weeks or months of animal loading, if done non-stop 24 hours a day. If the loading was all done in one week as the Bible says you would have to load two to three pairs a second non-stop for 7 days at the slowest.

Answer: Loading the ark is only a time constraint if one insists on there being such a large amount of animals to load. This issue is again one of world view in which skeptics think that two of every animal today would needed to have been loaded on the ark. The difference being that the variety of animals today was not the same as the variety of animals present pre-flood.

Today we are surrounded by such a massive abundance of plant and animal variety, which in reality came from a more narrowed down source. For example, 500 varieties of sweet peas have come from one singe type.[9] There are over 200 breeds of dogs today, despite all coming from one common wild dog (wolf, dingo, etc.)[Figure 2].[10] The wide variety of pigeons we know of today all came from the wild rock pigeon from Europe.[11] So only a small percentage of the animals are needed to repopulate and produce the wide variety of animals we observe today. Then take into consideration that only land dwelling air breathing animals and insects were commanded to board the ark (since aquatic animals, even amphibians, could survive the flood in very small numbers) and you’ve significantly cut down on the number of animals that needed to board the ark.

Figure 2

So how many animals and insects needed to board the ark? Ernst Mayer, one of America’s leading systematic taxonomists lists that there are 1,072,300 species of animals.[12] But most of these species live in the seas. And of those land dwelling creatures remaining, most are very small, like insects. With all ineligible animals removed from the picture, we’re left with only 21,160 varieties of animals that would need to board the ark, or 43,000 coupled (total) at a max. When examining the types of animals more closely, baraminologist A.J. Jones estimates only 2,000 types of animals would need to board the ark.[13] John Woodmorappe suggests 16,000 animals would needed to have boarded the ark at the most.[14] The general consensus from creation scientists puts the number around 35,000 animals total boarding the ark.[15]

With all this considered the time taken for these animals to board the ark is no longer so significant. Given that the animals had one week to board the ark, such a feat could be accomplished fairly easily for eight adults. For example, if 16,000 animals needed to board the ark, in seven days that could be accomplished by loading 50 pairs an hour. At a max of 21,160 pairs, it would take 125 pairs an hour, or a little over two pairs per minute. All very plausible, especially when considering all the very small creatures which could simply be gathered.

Problem 5: Fitting All the Animals on theArk.There are millions of animal species in the world, most of which are extinct. It would therefore be impossible to enclose even a fraction of them on an Ark 450 to 510 feet long.

Answer: As stated in the last answer, fitting all the animals on the ark is not a problem if only 16,000 to 43,000 needed to be on the ark. The dimensions of the ark allow for as much as 145 million cubic feet or 41,000 cubic meters of room inside the ark.[16] Which is the equivalent of 270 or more railroad cars. Large animals like elephants or giraffes were most likely taken aboard as infants which would no doubt save space as well as food and waste constraints. And considering many animals were small birds, rodents or insects, it is possible for only one-fourth of the Ark’s interior would be used for 16,000 animals[17], only two-thirds used for the max 43,000 animals.[18] This of course would leave plenty of room for food, water, and Noah’s small family [Figure 3]. Of which, Woodmorappe proposes that only six to twelve percent of the arks interior would be needed to store food and nine percent would be needed to store water.[19]

Figure 3

Problem 6: Caring for the Animals. Where was all the room for the food? What about animals with specific diets? How was the food kept from spoiling? How was all that animal waste disposed of?  How did a handful of people take care of thousands of animals for a year?

Answer: Skeptics often compare the needs of animals at zoos to that of animals on the ark. Because of this inappropriate comparison, the impossibility of eight people feeding and cleaning up after so many animals is suggested. This comparison is inappropriate however, because zoos keep animals for long periods of time with a goal of keeping the animals healthy and comfortable. The ark however would only be keeping animals for a short period of time (one year) with the goal only being that of survival. This distinction is important, and it is therefore fallacious to apply zoo keeping conditions as mandatory conditions for the ark.

Now, taking into consideration there were only thousands of animals on the ark and not millions, it is much more feasible to propose a small group of people caring for them. Even at that, most skeptics point out the trouble of walking from pen to pen caring for the animals daily. The Bible however does not state how the animals were cared for. It is possible that mechanisms were used like self-feeders, water piped in from bamboo, or angled slats to direct waste to common areas where it could be disposed of.

Also, as said before, specialization of animals would have occurred after the flood, so specialized conditions and diets was most likely not an issue. Many animals eat specialized diets as we observe them today because of their distinct ecosystems in which they now live. Many animals however are not locked into a specific diet and can survive off alternative foods they never ate in the wild.[20] That is, alternative foods which could have been dried and kept in storage for long periods of time without spoiling.

Lastly, we cannot overlook animal hibernation. According the Encyclopedia Britannica, “…another mechanism used by some organisms to avoid stressful environmental conditions is that of dormancy, an inactive state accompanied by a lower than normal rate of metabolism- the chemical processes responsible for the activity, nourishment, and growth of an organism- during which an organism conserves the amount of energy available to it and makes few demands on it’s environment. Most major groups of animals as well as plants have some representatives that can become dormant.”[21] A wide variety of animals on the ark had the ability to hibernate, which would cut down on the amount of resources those animals consumed and the time needed to care for them. This is especially note worthy when dormancy within animals can be triggered by food shortages and changes to oxygen and carbon dioxide levels in their environment, which most definitely would have occurred during the global flood. With that, another question can be postulated: Were all or most of the animals hibernating? Though only a small percentage of animals on the ark could hibernate by today’ standards, more animals may have had the ability to hibernate in the past. We also should not overlook the possibility of God inducing dormancy supernaturally in the animals to aid in their survival while aboard the ark. It may even be suggested that this is the first time dormancy or hibernation was utilized by animals if the pre-flood conditions on earth did not require it. If all the animals had the capability to hibernate on the ark it is possible this capability was lost after the flood as animals became specialized in their post flood ecological niches.

Now skeptics might ask why Noah had to bring food for the animals if they all were hibernating. But this is first neglecting the fact that the animals had to be fed during the boarding process. And second, it shows a lack of understanding of hibernation in animals. Hibernation is not necessarily a deep sleep for months at a time in which the animals does not eat at all.[22] Hibernation observed in most animals is lethargic behavior, fluctuating periods of sleep, and periodical small feeding.[23] Because Genesis does not go into detail on this subject, there is a wide variety of solutions to the care taking “problem.” But it is definitely within the realm of possibility. Not impossibility.

THE FLOOD

Problem 7: Flood Water. Where did all the water come from? There isn’t nearly enough water on the entire planet to cover all of the earth including its mountain tops. After the flood, where did all the water go?

Answer: The Bible states in Genesis 7:11-12 that all the water came from above the earth (rain) and below the earth (fountains from the deep), which would have been driven by volcanic and geothermal events. The current creationist model suggests that the supernatural initiation of plate tectonics caused the disruption which caused the natural catastrophe which drove the global flood. As Dr. Snelling describes in detail;

“As slabs of oceanic lithosphere subducted, elsewhere the tension ripped apart the pre-flood ocean floor and produced rapid extension  along globe-encircling linear belts . This rifting of the oceanic crust  would have allowed mantle material to upwell, the partial melting of it producing new basaltic  ocean crust as sea floor spreading progressed. Because this rifting was so catastrophic and rapid, the pressure drop experienced by the upwelling mantle material was also rapid, causing many of the minerals to find themselves above their melting temperatures, which in turn would have resulted in rapid partial melting.  The hot magma rapidly rising to the ocean floor to cool as new oceanic crust would have come in contact with the ocean water, instantly forming superheated steam. Together with the volatiles degassing from the magma itself, the steam would have erupted from the V-shaped rifts on the ocean floor at super sonic speed as spectacular jets, rising high into the earth’s atmosphere. These jets would have entrained large amounts of liquid sea water and carried it aloft high into the atmosphere. Thus, there would have been a linear chain of geyser-like fountains along the tens of thousands of kilometers of globe encircling  mid-oceanic ridge, matching closely to the description provided in Genesis 7:11. The entrained water carried into the atmosphere would have then fallen back to the earth as intense global rain, which suggests that the opening of the “window of heaven” of Genesis 7:11 is describing the rather sudden onslaught of torrential rain from the water carried aloft by the globe-encircling fountains arising out of the great ocean depths.”[24]

So we have two sources from where the water originated. As far as there not being enough of it to cover the entire earth, that observation is based on how the earth is shaped today with deep oceans and high mountain ranges on land. But remember, prior to the flood the continents had not divided yet, so mountain terrains were not nearly as high as we observe today and the oceans may have been more shallow making it very possible for the water present today being sufficient to cover the entire globe at one time. After which, the flood waters receded into the seas and ocean basins as we currently observe today.

Problem 8: Tsunamis. How did theArksurvive all the Tsunamis? Surely the global flood would produce huge tsunamis. Tsunamis can cause massive destruction on modern day reinforced structures, how could theArkhave withstood not just one tsunami, but multiple tsunamis over the course of a year?

Answer: Tsunamis devastate coast lines because the massive amount of energy that drives the tsunami enters shallow waters and has no place to go but upward above the water forming massive waves or surges. However, over deep water this energy has more room and therefore does not cause hardly any surface disruption.  A tsunami in deep water will usually create a very fast wave about one foot high.[25] The ark would of course be floating over very deep water and would not be subjected to massive tsunamis.[26]

Problem 9: Oxygen and warmth. With the water rising over all the mountain tops, how did the animals on the ark stay warm and breath?

Answer: Atmospheric pressure is responsible for the conditions found on high mountain tops. The higher you go, the less atmosphere you’re under which is why there is a lack of oxygen and colder temperatures. This however would not be an issue for the Ark because atmospheric pressure is related to the sea level.[27] If the sea level rises, and the ark is floating on the sea, then the passengers on the ark will always experience oxygen levels and temperatures found at sea level, not high mountain tops.

Problem 10: Ice Core Evidence. Ice core evidence fromGreenlandgoes back 40,000 years. Some as far back as 120,000 years. But if the Flood was less than 10,000 years ago, the ice sheets of Greenland would have been altered or destroyed by such a flood and therefore could not be 40,000 or 120,000 years old.

Answer: The problem with ice core samples is that their ages are assumed, not fact. It is assumed ice core samples go back a hundred thousand years because it is assumed that the ice sheet formation is very slow and gradual. Case in point is the “Lost Squadron” from WWII. In 1942, a flight of P-38 fighter planes and B-17 bombers were flying to Greenlandwhen a blizzard forced them to crash land on the ice. The crews were rescued but the planes were damaged and left behind. In the 1980’s plans were made to locate these planes and see if they could be salvaged. Based on the general assumed notion that the ice sheet formation is very slow, and only forty years had elapsed since the planes landed on the ice, it was believed that the planes would have maybe a few inches of ice or slush on them. Finding the planes was very difficult however, and about ten years later they were finally located… under 250 feet (75 meters) of ice![28] So much for a slow and gradual formation of ice! Then consider that the ice sheets inGreenland average 300 meters. So if planes from 1942 were found fifty years later under 75 meters of ice, then it is clear that the slow and gradual formation of ice sheets is not correct. And thus they could have formed in thousands of years, not a hundred thousand years. For a more technical study on how ice sheets form and are dated I recommend visiting this site: http://www.answersingenesis.org/articles/tj/v15/n3/greenland

Problem 11: Relative Mountain Ages. For example, why weren’t the Sierra Nevadas eroded as much as theAppalachiansduring the Flood?

Answer: This “problem” assumes that both mountains were in existence prior to the flood, or created at the exact same time during the flood and should therefore have the same erosion features. However, the plate tectonic movement would have created the Appalachian mountain range first exposing the range to more erosion from the flood than the later formed Sierra mountains. As Dr. Snelling describes, “The progressive cycle of catastrophic plate tectonics would ensure, as continental fragments collided, these strata sequences were buckled and folded, faulted and uplifted, perhaps to be temporarily above the Flood waters, only to be eroded again as the flood continued.”[29] As evidence; within deeply eroded mountains like the Appalachians (and the Himalayas or European Alps), exist high-pressure, low-temperature minerals in rock core samples taken from these mountains. Such minerals would be expected in the event of an accelerated geological process which would occur with these mountains rapidly deforming, being buried and then eroded.[30]

Problem 12: Sea Floor Evidence. A year long flood should be recognizable in sea bottom cores by (1) an uncharacteristic amount of terrestrial detritus, (2) different grain size distributions in the sediment, (3) a shift in oxygen isotope ratios (rain has a different isotopic composition from seawater), (4) a massive extinction, and (n) other characters. Why do none of these show up?

Answer: The problems listed here are solved by simply understanding the creation model of the Flood. (1) No large amounts of terrestrial detritus would be expected to be found in the sea floor because of cataclysmic upwelling of the mantle which lifted the ocean lithosphere, forcing the ocean waters onto the land, which is why there are numerous evidences of sea life remains found on land today. In other words, the transportation of water and accumulation of sediment was land bound not ocean bound during the flood, which is why there is no terrestrial evidence in the ocean today. (2) Different grain size distribution (which is evidence of rising or receding waters), would also not be expected in the sea floor since sediment was deposited over the land. Interestingly enough, the sediments that are deposited over land do show rising water levels much higher than the ocean’s current level today. Seismic stratigraphy records show a rise in water levels above current ocean levels starting in the Cambrian deposit that never receded back to current levels until after the Tertiary layers were deposited.[31] (3) The oxygen isotope ratios are solved again based on the flood model depositing sediment over the land, not in the ocean. Which, ironically, oxygen isotopes are evidenced as supporting the flood model as oxygen 18 and oxygen 16 ratios found in rock units and shells gradually increase from the Paleozoic and Mesozoic strata of the geological record. These increases are evidence of heating, at which the ocean would have increased in temperature as the magma burst through open spreading centers in the ocean as the mantle rose pushing up the ocean lithosphere.[32] (4) As for the massive extinction? Again, the sediments were transported over land, not the ocean basins, so no fossilized extinctions would be expected in the sea floor.

Problem 13: Tree Rings. Why is there no evidence of a flood in tree ring dating? Tree ring records go back more than 10,000 years, with no evidence of a catastrophe during that time.

Answer: This believed “problem” already assumes that the flood did not happen thousands of years ago, or that if the flood occurred thousands of years ago and we have tree rings that go back further, then trees survived the flood and therefore their rings should show evidence of it. But a global flood would have destroyed the vast majority of trees, fossilizing some. The post-flood trees would therefore not have been flood survivors but instead new seed sprouting. So we should not expect to see evidence of a flood in tree rings.

The problem is the misconception of tree rings going back 10,000 years or more. This is a highly debated subject even in the secular realm. Tree ring records are not perfect records to go off of. Trees can produce multiple tree rings in the same year. In observed cases trees can grow up to five rings in the same year.[33] It has also been observed that tress growing side by side will produce a different number of tree rings.[34] In addition, tree ring size (whether large or smaller) can be influenced by dramatic changes in climate, which we would expect after the flood. Comparing tree rings from tree to tree often results in inconsistencies when rings sizes are not similar, and usually leads to long ages like 10,000 years when trees are cross examined with each other.[35] This, again, is a chronological assumption, and it is a poor argument to make against the flood. According to agricultural and plant scientist Dr. Don Batten, “The extended tree ring chronologies are far from absolute, in spite of the popular hype.”[36]

Problem 14: Too Much Heat. The subduction process and rapid plate tectonic movement would release so much heat energy it would boil all the water in the ocean.

Answer: Actually, boiling the water in the ocean is exactly what the flood model suggests, but only in concentrated areas. The heat released as the “fountains of the deep” opened would create super heated steam to rise to exceedingly high levels in the atmosphere producing all the torrential rain fall. The energy released from all this plate activity is estimated to be in the area of 1028 joules.[37] Many argue that such energy is actually required to form many geological features found on earth that cannot be reasonably explained by the uniformitarian low energy, slow and gradual plate movement. For example, continental margin collapses that have moved mountain-size blocks of strata tens of kilometers,[38] or high pressure produced minerals (normally found deep within the earth’s surface) located in collision zones within mountain belts.[39] Massive catastrophic geological activity, as the flood model suggests, are required to form such features.

Problem 15: Cenozoic Sediment. Cenozoic sediments are post-Flood according to the runaway subduction model. Yet fossils from Cenozoic sediments alone show a 65-million-year record of evolution, including a great deal of the diversification of mammals and angiosperms.

Answer: Just because the flood may have ended does not mean that catastrophism ended as well. The massive changes made to the crustal thickness produced during the flood would leave the earth in isostatic disequilibrium with rocks buried very deep in the earth’s interior.[40] These rocks would no doubt become subjected to enormous pressure and temperature, melting them into less dense magmas that would rise to the earth’s surface where some would cool under the surface as intrusions and others form volcanoes in certain areas. The post-Flood environment would be subject to a lot of volcanic activity where large amounts of volcanic ash and debris flows would bury and preserve fossils which would account for the Cenozoic sialic volcanism evidence.[41] Also present in the post-Flood environment would be heavy rainfall in the first few years as the oceans continued to release their heat creating massive storm systems. This heavy rainfall would create catastrophic sedimentation despite occurring after the flood. The eroded sediments would (and did) wash into lakes depositing and preserving millions of different fossilized creatures like fish, frogs, turtles, lizards, snakes, alligators, birds, bats and invertebrates as has been observed in Green River Formation.[42] Similar fossil bearing Cenozoic sedimentation deposits are found in lakes inWyoming,Utah andColorado. This would explain the presence of fossil bearing sediment after the flood, but it being 65 million years old is an evolutionary assumption which will be refuted in a later addressed problem.

Problem 16: High elevated sediments. Why are most sediments on high ground? Most sediments are carried until the water slows down or stops. If the water stopped in the oceans, we should expect more sediments there. Yet sediments on the ocean basin average 0.6 km thick, while on continents (including continental shelves), they average 2.6 km thick.

Answer: As said before, the sediments were deposited over land, not over the ocean basins during the flood. Since then erosion has carried sediments into the ocean, but not nearly as much as the flood did over land. Additionally, as the ocean plate subducts under the continental plates, since it is lower in density, it often crinkles or pushes up the continental strata and subducts much of the sea floor. Simultaneously the continental plate scrapes up the sea floor surface, both of which increase the continental plates sediment thickness when compared to the ocean sediments. Lastly, it should not be overlooked that such thin sedimentary deposits in the ocean and thick sedimentary deposits high on land work against the uniformitarian assumption of slow and gradual process. Twelve studies have concluded that the volume of sediments eroded from earth’s surface and deposited in the oceans is about 24 billion metric tons per year.[43] Through the process of subduction, the ocean surface only takes 1 billion tons of sediment down with it per year.[44] It doesn’t take a math wiz to figure out that this would lead to 23 billion tons of sediment settling down into the ocean per year. At this rate the entire ocean would become completely clogged with sediment in just 12 million years.[45] On land erosion rates should have completely eaten away at any exposed sedimentary layers long ago. This is because erosion rates on earth average 2.4 inches per thousand years.[46] At this rate, the North American continent should be flush with the sea level after 10 million years of this erosion![47] Most geologists say this is not an issue because of the constant renewing of the surface material from below via uplift forces. But this doesn’t account for the large amount of sediment still present that hasn’t eroded away. If earth is as old as we’re told it is, the land should be stripped of sediments which would have settled in the ocean, successfully clogging the entire ocean… but instead the exact opposite is true…

Problem 17: Consistent geological dates. If the geological dates assigned to the geological record are not correct, then how come radiometric dating just so happens to coincidentally match up perfectly? Surely this is not a mere coincidence over and over again…

Answer: Refuting this claim actually requires a large body of work, which for sake of space I will try to summarize as briefly as possible. To start, it should be recognized that radiometric dating cannot date everything in the ground. It is mostly used on igneous rocks like granite or basalt which were once magmas that cooled and solidified into rock.[48] It is rarely used on sedimentary rocks which are comprised of preexisting materials and rocks redeposit in another location. When a rock cools it can sometimes contain radioactive isotopes that are very unstable, disintegrating over time. The isotope’s original form is called the “parent” isotope, and what it ends up breaking down into is the “daughter” isotope. Therefore, when an igneous rock is analyzed, by measuring the ratio of parent to daughter isotopes, one can determine how old the rock is. It sounds fool-proof, but it is based entirely on assumptions.

Although the isotope ratio currently within the rock can be observed and measured, it cannot be known the conditions from which the rock formed and the environment the rock was placed in since then. There are essentially three factors that scientists assume with radiometric dating: (1) The initial ratio of parent and daughter isotopes in the rock when it first cooled or crystallized. It is assumed that there is no daughter isotopes within the rock initially, and therefore any daughter isotopes found within the rock are assumed to be the result of decay. (2) The ratio of parent and daughter isotopes after crystallization has not been altered since, except by decay. In other words, since formation the rock it could neither gain nor lose any isotopes except through radioactive decay. (3) The rate of decay has always been the same and has never changed.[49] Think of it like an hour glass. Imagine you walk into a room and find an hour glass draining sand from end to the other. How long has it been since that hour glass was turned over? You may look at the hour glass and notice that the sand in both ends appears equal so it has probably been draining for 30 minutes. But do you really know that. How do you know there wasn’t already sand in bottom when it was turned? How do you know someone hadn’t removed or added any sand to the hourglass wile it was draining? How do you know the sand has been falling at a constant rate? Since you were not there to see the hour glass turned and monitor it ever since, you cannot know for sure. Such is the case with rock formation.

What we can do is test rocks against each other. Put the same rock to multiple tests. Or simply put, take actual rocks we’ve witnessed crystallizing to be tested. Heck, if radioisotope dating is accurate, rocks should accurately test against one another, one rock should get the same age despite multiple tests, and the dating should match the time we saw certain rocks actually form. But such is not the case. Rock samples from the freshly formed lava dome from the Mount St. Helenseruption went in for Potassium-Argon dating and came back with an age range of 0.5 to 2.8 million years old[50]… but we saw the rock form in 1986. Rocks taken from New Zealand’s active volcano Mount Ngauruhoe were known to be less than 70 years old, yet when sent in to get dated they yielded an age range from 0.27 to 3.5 million years old.[51] One experiment involved taking multiple rock samples from different locations (to omit bias) and sending one sample from each location to multiple labs. Each lab produced different ages for the same rocks with some differences being up to 500 million years.[52] Rock samples from “old” strata and “young” strata of the Grand Canyon produced results that aged the “old” rock at 1.11 billion years old and “young” rock at 1.14 billion years old.[53] How does that make sense? Samples from the Bear Tooth Mountains produced an age range of over one billion years for the same rocks, and Bass Rapid Sill samples produced an age range of 537 million years for the same rocks.[54]

In summary, there is nothing “consistent” about radiometric dating and the geological record. They DO NOT match up perfectly, as we’re falsely lead to believe. The only coincidence here is that a dating method that is based on unknown assumptions provides contradicting ages, an enormous range of ages, and most importantly, flat out WRONG ages.

Problem 18: Fossil sorting. How come the sorting of fossils in the fossil record is perfect for evolution? Why don’t we see more modern animals like elephants fossilized along side older animals like dinosaurs? Or older plants alongside newer plants?

Answer: Stating that fossil sorting is “perfect” for evolution is an assumption that evolution is fact to begin with. After all, the fossilized layers in the ground are only perfect for evolution if you believe the fossilized layering is the result of slow and progressive evolution. How is that not circular reasoning? One could, however, examine this same layering and conclude it is evidence of global flood activity.

The first concept that needs to be established is that the fossil record is not a record of “life” but a record of “death.” Massive and catastrophic death. Even evolutionists believe that the fossil record is a record of 17 massive extinction periods,[55] they just believe there were millions of years of evolution between these periods of extinction and fossilization. The creationist on the other hand believes that the majority of the fossils stem from one single catastrophic event; the Flood. In addition, animals found fossilized next to each other didn’t necessarily live together, but they certainly did die together.

So how could the fossil record be interpreted as evidence of the flood? Well, all one must do is observe the variety of ecosystems worldwide. We have deserts and alpine forests and tropical rainforests and artic tundra. Some ecosystems bear millions of species, some seem to bear hardly any at all. But they most certainly are very different from one another in the variety of creatures that live within them. Such ecological distinctions were present in the pre-Flood world (though not as distinct or extreme as today’s environments) in different regions, but most importantly, different altitudes. As the flood waters rose they would inundate, wash away, and then bury such zones sequentially. So creatures found in the lower strata of the geological record would be creatures that lived at lower altitudes and warmer climates, while those found in higher strata lived in higher altitudes and colder climates.

So to break it down, the Flood can account for sorting of fossils as such: As the ocean lithosphere began to break up in the beginning of the flood and rise, massive currents would have swept up the sea floor sediments depositing them over land constituting the primitive organisms like brachiopods, bivalves, gastropods, corals, graptolites, echinoderms and crustaceans, that lived in the sea found in the lower strata.[56] Eventually these currents would hit shallower ocean waters consuming the life there. It should be noted that the lower strata of the Cambrian, Silurian, Ordovician and Devonian periods are almost exclusively a record of shallow marine invertebrates with fish and amphibians only found in the higher Ordovician and Devonian strata.[57] According to French Geologist Dr. Maurice Gignoux, “At least 1500 species of invertebrates are known in the Cambrian, all marine, of which 60% are trilobites and 30% brachiopods.”[58] In fact, it is not until the higher Carboniferous and Permian strata that we come across the first land animals.[59] Take into account behavior and higher mobility of vertebrates over invertebrates, and it becomes obvious that vertebrates had the mobility and capability to try to find safety on higher lands as the flood waters rose. The higher up in the strata the more capable the animals were to evading the rising waters. Humans, being the most capable and adaptable to avoid the flood waters would of course be last in the sequence. Additionally hydrodynamic sorting, which will be later covered, would have also lead to significant sorting of the fossil record.

Dr. Snelling summarizes, “Because the Flood began in the ocean basins with the breaking up of the fountains of the great deep, strong and destructive ocean currents were generated by upheavals and moved swiftly landward, scouring the sediments on the ocean floor and carrying them and the organisms living in, on and near them. These currents and sediments reached shallower continental shelves , where shallow-water marine invertebrates lived… Unable to escape, these organisms would have been swept away and buried in the sediment layers as they were dumped where the waters crashed onto the land surfaces being progressively inundated farther inland… the sediments washed shoreward from ocean basins would have progressively buried fish, then amphibians and reptiles living in lowland, swampy habitats, before eventually sweeping away the dinosaurs and burying them next, and finally at the highest elevations destroying and burying birds, mammals and angiosperm.”[60] So as you can see, the sorting of fossils that is claimed to be so convenient for evolution, can be just as convenient for the creation model of the flood. As far as why elephants aren’t buried with dinosaurs or “newer” plants with “older” plants. If such organisms lived in different regions or elevations and had different capabilities for escaping the rising water, then they would be subsequently inundated and buried by the flood sediments at different times. Lastly, it should be noted that the presence of “living fossils” found today despite being thought of as extinct for millions of years, living beyond the age of rock they were once exclusively found in, throws a wrench in the fossil sorting for evolutionists, but supports creationists

Problem 19: Fossils under coral reefs. How did fossils end up under coral reefs hundreds of feet thick and miles long?

Answer: The supposed problem here is that coral reefs grow very slowly over thousands of years, which naturalists date back prior to the flood. So how could fossils created by the flood be found under reefs that would have taken thousands of years to grow themselves? The problem is in the assumption made by this question. The assumption is drawn from coral reef “pieces” that are found, not an actual intact coral reef hundreds of feet thick and miles long.

Geologist Dr. Emil Silvestru explains, “When pieces of bioherms have been found inside masses of carbonate they were assumed to reflect a reef. In fact, modern reefs are never incorporated in carbonate platforms (the vast majority of limestone deposits in the rock record are areal platforms [those covering a large area rather than a localized deposition] and even larger surfaces. When preserved as narrow limestone ridges with broken elements of reef, limestone massifs are almost always olistoliths [very large chunks of rocks resting on younger rocks] revealing tectonic displacement. They frequently rest on ophiolites [basalts placed along mid ocean ridges], not fossil beds.”[61]

In conclusion, what is claimed to be coral reefs, aren’t coral reefs at all but merely pieces of coral reefs. There is therefore, no conflict with the flood model.

Problem 20: The floating and sinking of organisms. Why are smaller organisms found in the lowest strata when they are lighter and would have taken longer to sink? They should be found in higher strata.

Answer: If smaller organisms took longer to sink as they slowly settled down to the bottom there would be no fossils of them, period![62] Fossilization is a rapid burial process. Therefore, they should not be found in higher strata for sinking slowly. There is also hydrodynamic sorting to consider. As Dr. Snelling explains, “Moving water hydrodynamically selects and sorts particles of similar sizes and shapes. Together with the effect of the specific gravities of the respective organisms, this would have ensured deposition of the supposedly simple marine invertebrates in the first-deposited strata that are now deep in the geologic record of the Flood… Moving water, or moving particles in still water, exerts ‘drag’ forces on immersed bodies… Particles in motion will tend to settle out in proportion mainly to their specific gravity (or density) and sphericity.”[63] Despite the marine organisms being smaller they were very dense for there size. Trilobites and brachiopods for example are mostly composed of calcium carbonate and calcium phosphate, which are dense, heavier than quartz, which is the most common rock found in sands and gravels.[64] Taking into consideration the first creatures impacted by the Flood would have been the smaller sea organisms and it is easy to understand why they appear first in the fossil record.

Problem 21: Footprints. Why are foot prints sorted in geological layers? Shouldn’t they be found in only one place?

Answer: Interestingly enough, most animal tracks imprinted on the slopes of under water sand dunes are traveling uphill.[65] Naturally as animals were being swept up by the flood those that were temporarily fortunate enough to be washed into areas where they could get a footing would obviously scramble up to higher elevations. As the strata was deposited periodically, there would also be evidence of animals scrambling out of the flood waters in the deposits, only to be eventually consumed by increasing flood waters. This would explain why there are various footprints through out the record.

Problem 22: Human artifacts. Why don’t we find human artifacts mixed in with other extinct animals. If at the time of the flood, humans were coexisting with all these animals shouldn’t we find tools, buildings and other man made objects along side trilobites for example?

Answer: We wouldn’t find trilobites with human artifacts simply because trilobites lived on the ocean floor and last time I checked humans did not live on the ocean floor. Human artifacts like buildings and other constructed objects would have surely been destroyed by the flood waters, with their materials, if stones or brick, scattered and indistinguishable. Any materials made of wood would have floated and would not have been caught in the fossilization process which also explains the lack of human fossils with other animals fossils as humans would have sought shelter on floating objects as well as have been more capable of reaching high ground, thus eliminating any significant chance of humans being rapidly buried and subsequently fossilized.

Problem 23: Ecological sorting. Ecological information is consistent within but not between layers. Fossil pollen is one of the more important indicators of different levels of strata. Each plant has different and distinct pollen, and, by telling which plants produced the fossil pollen, it is easy to see what the climate was like in different strata. Was the pollen hydraulically sorted by the flood water so that the climatic evidence is different for each layer?

Answer: Ecological sorting would be consistent with the creation model asserting that the flood waters reached different elevations periodically accounting for the differences in ecological environments. But using fossil pollen for ecological system indicators is not appropriate. As Dr. Silvestru explains, “Pollen-based climate reconstructions are viable only for the Quaternary. We have no idea how the pollen of extinct plants can reflect climate. Other data (usually oxygen isotopes) is used for that. Previous to the Flood, there was a vast array of plants and pollen, and the climate was mild worldwide. Fossil pollen actually reflects that. Only in the Quaternary, because of the Ice Age, do we have drastic flora shifts recorded by pollen. Well, the Quaternary is post-Flood.”[66]

Problem 24: Surface features below surface. How can surface features like, up-right trees, wind blown dunes, river channels, beaches, animal burrows etc, appear in geological layers far below the surface if they at one time were not the exposed land surface?

Answer: This problem is similar to the footprint “problem.” If it is understood that the flood waters deposited sediments periodically such features are possible. Additionally, some of the above mentioned examples like erosion and burrows are rarely found between sedimentary layers,[67] which should in fact be the opposite if the sediments were laid down slowly and gradually over millions of years. There should be much more evidence of such features if that was the case but they are rare in the sense that millions of years of time should have left a staggering record of such activity. This is a problem for conventional geologists.

A perfect example is a case in Venezuelawhere two thin coal seams were separated by 30 cm of clay. The coal seams were dated to the Paleocene and upper Eocene, which are periods separated by a supposed 15 million years.[68] So in 15 million the only thing we have to show for it is 30 cm of clay which have no evidence of 15 million years of erosion or biological activity…  In the United States there is a massive formation from the Pliocene (2 to 5 million years old) that sits directly on top of the Triassic Trujillo Formation (208 million years old).[69] So we have two formations right on top of each other despite a 200 million year period separating them. TheTrujillo formation is very soft and there should 200 million years of valley and canyon erosion in it. But instead it sits flat and flush below the Pliocene formation.

Geologist Edwin McKee of the US Geological Survey wrote, “In a number of areas no physical evidence of a physical break has been detected and at these places a boundary between formations can be established only by placing it arbitrarily where a lithologic change occurs. Thus, the significance of the surface as a record of regional erosion seems questionable.”[70]

Yet there are still features (as few as they may be) which to the average skeptic should not be present in the event of a sudden global flood. Trees being found between strata will be discussed in a later problem (#29). But as for wind blown dunes all that one has to do is ask them selves how do they know the dunes are wind blown. This is a very common misconception when sand dunes are found in cross beds. It is therefore assumed the surface was once a dry desert landscape, which would not be possible during a global flood. But studies conducted on footprints found in these “desert” sand dunes indicate some were formed on sand under water and some along the water’s edge.[71] A great example of supposed windblown dunes is the deposit in the Coconino Sandstones of the Grand Canyon. Interestingly enough, all but one of the fossilized track ways found in the Coconino dunes were that of vertebrates traveling up slope and the direction of travel was different than the direction their toes were pointed in.[72] This would only be possible if the vertebrates were being moved along by a current underwater while trying to scramble up slope out of the flood waters. In addition, today it is often observed that large storms or tides create underwater dunes in sea floor sands called “sand waves.”[73] These underwater sand dunes have a high quartz content just like those found between strata. Sand waves also have shallow slopes whereas sand dunes found in the desert have steeper slopes. Interestingly enough the sand formations between strata have shallow slopes like that of sand waves.[74] Additionally, sand grain sizes measured in the sand features between strata match more closely to that of sand waves than wind blown sand dunes.[75] And the last bit of evidence proving that these formations between strata were underwater sand waves and not dry windblown sand dunes is the presence of a feature called “parting lineation.” Parting lineation only forms in fast-flowing waters and has never been observed in any desert sand dunes.[76] With all this considered, it is easy to conclude that the sand dunes found are not that of dry wind blown dunes, but of submerged flood driven sand waves.

Here you can see the submerged sand waves found near San Francisco California in the bay surface.

When considering the river channels discovered far below ground within the strata, they are consistent with fast flowing debris moving channels as we would expect in a global flood, not meandering slow moving rivers with sediment build up as would be the case with a surface river over the course of millions of years. Animal burrowing, though sparse, would be expected to as animals capable of burrowing would attempt to burrow up and out of the depositing flood sediments. Over the course of millions of years we should find a much larger concentration of animal burrowing evidence, which we do not. Beaches would also be present as flood waters deposited sand periodically. So clearly all the claimed surface features found deep within strata do not require millions of years to be there, and as the evidence shows, more than likely occurred during the flood.

Problem 25: Angular unconformities. How does a global flood explain angular unconformities? These are where one set of layers of sediments have been extensively modified (e.g., tilted) and eroded before a second set of layers were deposited on top. They thus seem to require at least two periods of deposition (more, where there is more than one unconformity) with long periods of time in between to account for the deformation, erosion, and weathering observed.

Answer: Angular unconformities, like the Great Unconformity found in theGrand Canyon, are definitely considered in the Biblical model for Earth’s geology. If one knew the creation model they would know that the Precambrian rock forming the angled unconformity is believed to have been created mostly in the initial creation week, Pre-Flood. Hence it’s vast lacking of fossils. Whereas the Paleozoic rock was deposited during the flood. Unconformities in between are possible too considering the sedimentary deposits were sequential and not a simultaneous deposit in mass. This would account for the “two periods of deposition” with erosion in between. Erosion, mind you, that is more applicable to catastrophic forces than slow and gradual forces.

Problem 26: Granite batholiths. When did granite batholiths form? Some of these are intruded into older sediments and have younger sediments on their eroded top surfaces. It takes a long time for magma to cool into granite, nor does granite erode very quickly.

Answer: There are three factors necessary for granite batholiths: Generation, in which part of the continental crust in melted; Segregation, the now melted rock collects together; and Ascent, in which the now warmer and more buoyant melted rock ascends to the surface.[77] As the melted rock ascends it gradually cools off and erosion above the granite exposes it at the surface. Such a process no doubt seems as though it would require millions of years to occur…

However more recent studies on granite formation has lead to a new understanding of this process. It has been modeled that during the segregation phase veins of melted granite flow through porous fractures referred to as dykes. As the melted rock flows through these dykes they become wider and wider until they reach a critical point and collapse, the buildup of melted granite unable to flow further reaches a point to where another vein rapidly forms until it too becomes too large and collapses.[78] Thus the cycle repeats itself in rapid catastrophic fashion. Evidence of this is believed to be observed in granites which the extraction of the melted rock was so rapid the zirconium inside did not have time to equilibrate. Such is the case with granites from the Himalayas which were extracted in less than 150 years[79] and granites from Quebec which extracted in just 23 years.[80]

As far as ascension, old models suggested that granites ascended in one large mass upwards. But there were numerous problems with this notion, namely how such a large plume could push its way through to the surface. It’s like trying to push Jell-O through a brick wall. In addition the rising would be so slow that eventually the plume would cool and stop ascending (or “freeze”) never getting near the surface. But in more recent times it is now suggested and supported that melted granites rise through narrow dykes or along fault lines up towards the surface in rapid fashion pluming at the top. Granites in Coloradoand British Columbiasupport this notion, as they contain epidote crystals which is only possible if the granite ascended at a rate of 0.7 km to 1.4 km a year.[81]  Geologists estimate that a 6 meter wide gap is enough to transport melted rock upward at a rate of 1cm a second.[82] This rate is so fast that a narrow dyke 10 km long could supply enough melted granite in 350 years to form the entire 6,000 km2 Cordillera Blanca batholith inPeru.

Cooling times for the granite is not a long process either. A 2km thick plume of melted granite can cool in about 3,500 to 5,000 years.[83] So in summary, with evidence of rapid extraction, ascension and cooling it is entirely possible for granite plumes initiated by the flood’s geothermal activity to rapidly ascend to the surface where erosion exposed their surfaces. As Dr. Snelling asserts, “Even the conventional long-ages geologic community now regards the formation stages of granite plutons, after partial melting of source rocks to form granitic melts, that is, melt segregation, ascent and emplacement, to be ‘geologically extremely rapid—perhaps even catastrophic.’”[84]

Problem 27: Detailed Layering. How can a single flood be responsible for such extensively detailed layering? One formation in New Jerseyis six kilometers thick. If we grant 400 days for this to settle, and ignore possible compaction since the Flood, we still have 15 meters of sediment settling per day. And yet despite this, the chemical properties of the rock are neatly layered, with great changes (e.g.) in percent carbonate occurring within a few centimeters in the vertical direction. How does such a neat sorting process occur in the violent context of a universal flood dropping 15 meters of sediment per day? How can you explain a thin layer of high carbonate sediment being deposited over an area of ten thousand square kilometers for some thirty minutes, followed by thirty minutes of low carbonate deposition, etc.?

Answer: As it has been demonstrated in other answers, the flood was catastrophic. Enough to deposit large amounts of sediment. Hydrodynamic sorting, testable and observable, reveals the sorting principals of water currents. Neatly sorted sediments are found in the aftermath of hurricanes and flashfloods due to hydrodynamic sorting. More on this, as well as the high and low carbonate layering will be discussed further in the next problem of Varve Formation.

Problem 28: Varve Formation. The Green River formation inWyomingcontains 20,000,000 annual layers, or varves, identical to those being laid down today in certain lakes. The sediments are so fine that each layer would have required over a month to settle.

Answer: The thinly laminated shales known as varves (or rhythmites) are poor references for dating. It is conventionally thought they form during winter and summer sedimentation. But varve formation can vary due to flood discharges, climate changes, and differences in organic accumulation, all of which would vary the appearance and size of the varve deposits.[85] Such variations make the use varve deposits for dating troublesome, because there are too many variables that determine their make up which are not consistent, especially over long periods of time. For example, the Walensee of Switzerland generally receives an average deposit of two laminae a year, but has on many occasions received up to five laminae in one year because of flooding.[86] This unreliability is further indicated when varve counts are checked against radiocarbon dating and yet never seem to match. In one case a sequence of varves yielded an age of 28,000 years and radiocarbon dating of the same sequence yielded 10,000 years.[87] Sometimes the varves are so thin they are left undefined or are assumed missing, further indicating their unreliability.

Geologist Dr. Richard Flint writes of this uncertainty, “…the correlation of rhythmites, as described above, depends on the judgment of the person who matches the curves, and therefore is not wholly objective.”[88] He also has written, “Even the varve correlation… through the very short distance between Denmark and southern Sweden was severely criticized on the grounds that the implied relative dates of the several Danish deposits concerned are in complete conflict with the stratigraphic evidence. The whole matter of the reliability and usefulness of the varve correlation is at present in an unsatisfactory state.”[89]

But even if varve sequences are unreliable for gauging exact time, uniformitarians believe they still require slow and gradual accumulation to form. Too slow to only be a few thousand years old. In the event of large scale catastrophic flood, which could produce them quickly, skeptics point out that such finely laminate varves cannot be formed in sudden violent conditions. Yet, there are many modern observations of just that. In 1960, a hurricane flooded Floridaleaving behind a six inch layer of mud with numerous thinly laminated varves.[90] A flood in Colorado’s Bijou Creek deposited over 100 thinly laminated varves in just twelve hours.[91] But the greatest example came from the 1980 eruption of Mt. St. Helens which left behind a 25 foot thick layer of laminated ash in a matter of seconds.[92]

The thin laminae layed down at Mount St. Helens in a matter of seconds.

Lastly, the Green River formation in particular cannot be evidenced as incredibly slow accumulating annual varves because the varves are interbedded with various fossils, most notably catfish.[93] These fossils are in incredible condition with skin, scales and other “soft parts” still intact. Such preservation is only possible with rapid burial and fossilization. Slow and gradual varve development would not yield such preserved fossils… or any fossils for that matter.

Amazing preserved fish fossils found in the Green River formations.

Problem 29: Layered Fossil Forests. Stratigraphic sections showing a dozen or more mature forests layered atop each other–all with upright trunks, in-place roots, and well-developed soil–appear in many locations. One example, the Joggins section along the Bay of Fundy, shows a continuous section 2750 meters thick (along a 48-km sea cliff) with multiple in-place forests, some separated by hundreds of feet of strata, some even showing evidence of forest fires.

Answer: It is important to note, when considering fossil forests, that both vertical and horizontal tree stumps are found, exposed petrified trees have broken roots, intact roots are small root systems, not large root systems (those with large root systems have been broken off), the majority of the trees are cut off at the top and do not extend into higher strata where other petrified trees are located, and the few that do, do not show signs of rotting, weathering or insect infestation which would be evident if they were exposed in the open for a long period of time.[94] All of which is sufficient evidence that the fossilized trees were not fossilized where they originally grew but were instead transported elsewhere, deposited and then buried. Some object that fallen (horizontal) trees are present in today’s forests and therefore should not be considered evidence of a rapid and violent disturbance. But interestingly enough, all horizontal trees are orientated in the same direction, which is not evident in any of today’s forests.[95] Such uniform orientation of horizontal trees reveals a strong directional current, whether a volcanic mudslide or flood current, which would be present during the Flood. Analysis of the wood from the trees indicates they grew in the same environment, which would be the case in the event of a flood deposit, but would not be the case if the forests were separated by long periods of time.[96] In addition, many fossilized trees have been found standing upright through multiple layers of sedimentary strata, known as polystrate fossils.[97] This actually would testify to rapid depositing, not slow and gradual depositing as the tree would decompose long before the next deposit if it had taken long periods of time to form, nor can a tree grow through solidified sedimentary strata. The only plausible explanation for polystrate fossilized trees is rapid formation as would occur during the flood.

A superb example of such catastrophic forces causing just such a scenario as we find with fossilized forests is again Mount St. Helens. Millions of trees within 150 square miles of forest were blown down by a blast of superheated gas. This was followed by an avalanche from the summit as all the snow and ice on it instantly melted creating a tsunami of debris that pounded down into SpiritLake. Many trees from the blast zone were carried off into the lake from these forces where a number were grounded on the bottom with their roots buried in mud and organic debris. Some time later as the lake bottom was investigated many upright trees were found at the bottom as if they had grown in place there.[98] Many trees that were floating on the surface became water logged and eventually sank (root end down because the roots are denser) where they too took anchor in the accumulating sediment at the lake bottom. As time passed more and more trees sank and more and more loosely floating light sediments sank as well, providing a staggered layering of trees like we find in fossil forests today.

Now some of you may still be thinking that though this layering of trees is possible, petrification takes way too long to be a result of the flood. Yet, there is abundant evidence to the contrary. Petrification is a fast process. It is, after all, directly testable and repeated experiments testify to this. One such experiment conducted involved placing blocks of wood in hot alkaline springs to test the rate at which silica deposited on the wood cellular surface. The experiment yielded results of 0.1 to 0.4mm a year.[99] In Japan, full petrification of wood has also been observed to occur in tens to hundreds of years.[100] So clearly it is possible for fossilized forests to be the result of rapid catastrophic flood and volcanic activity.

Problem 30: Limestone Deposits. Much limestone is made of the skeletons of zillions of microscopic sea animals. Some deposits are thousands of meters thick. Were all those animals alive when the Flood started? If not, how do you explain the well-ordered sequence of fossils in the deposits? Roughly 1.5 x 1015 grams of calcium carbonate are deposited on the ocean floor each year. A deposition rate ten times as high for 5000 years before the Flood would still only account for less than 0.02% of limestone deposits.

Answer: The answer to this problem lies in how limestone deposits are formed. It is assumed that the accumulation of lime mud in tropical oceans is the only way such deposits can form. However, making such a comparison is not valid. The lime mud slowly accumulating in tropical oceans today are mostly comprised of silt sized crystals (90% aragonite and 10% calcite, about 20 microns in diameter), vs. the ancient limestone deposits we find in earth strata which are comprised of clay sized crystals of almost all calcite and a little dolomite, 4 microns in diameter) with larger sand sized skeletal fragments here and there.[101] As Dr. Randolph Steinen, head of the Connecticut Geological and Natural History Survey clarifies, “… the grain (crystal) size distribution and grain (crystal) shape characteristics of modern lime-mud sediment are very different from their lithified counterparts.”[102] Ergo, it would not be logical to compare the two and assert that ancient limestone deposits took just as long to form as we observe them now.

Additionally, the limestone deposits we study today must have lithified very rapidly because in them we have found some of the most preserved fossils, often called “perfect” fossils. One fish fossil from the Santana Formation in Brazilwas preserved so perfectly it is believed the lithification was instantaneous and that fossilization was probably the cause of death for the fish![103] The same perfect fossilizations are found in limestone deposits inGermany andMexico.

Lastly, such rapid settling of lime mud has been found possible in the Floridaand Bahamasregion. It was observed that in very calm waters the mud accumulated slowly. But in water with increased movement the mud aggravated into pelletoids with the hydrodynamic characteristics of sand grain sized particles. And thus, the mud settled very quickly![104]

Problem 31: Chalk Deposits. How could a flood have deposited chalk? Chalk is largely made up of the bodies of plankton 700 to 1000 angstroms in diameter. Objects this small settle at a rate of .0000154 mm/sec. In a year of the Flood, they could have settled about half a meter.

Answer: The first indicator that the chalk beds are not created as slowly as uniformists believe is that the chalk beds are pure, having nothing else mixed in with them.[105] This is difficult to believe considering how slowly they supposedly accumulate. In addition to that; many chalk beds are rhythmically bedded, have joint-like breaks or bedding planes, occasional thick marl bands, chalk pebbles within the marls, hollow erosion junctions, uneven distribution (not laid out flat), erosion on submarine surfaces of the chalk, and some even have fine-grained chalk with textural, parallel lamination bedding. All of which is evidence of rapid deposition and/or strong and turbid currents when deposited.[106] Thus, there is sufficient evidence that rapid accumulation occurred.

Furthermore, the planktons that create the chalk beds (coccoliths and diatomites) do not exist in consistent concentrations but instead fluctuate depending on conditions. When the conditions are right this can lead to “blooms” in which the plankton population explodes. It is known that blooms are connected to turbulent waters, wind, decaying fish, nutrients from freshwater, upwelling and temperature.[107] If you think about it, all the factors that drive these blooms would have been present during the cataclysmic flood in massive proportions. Surely there were massive blooms during this event. Most importantly, many types of plankton form large multi-cellular organisms several millimeters in length during these blooms which would have lead to their bodies sinking more rapidly to the bottom where they accumulated.

Problem 32: Salt Deposits. How could the Flood deposit layers of solid salt? Such layers are sometimes meters in width, interbedded with sediments containing marine fossils. This apparently occurs when a body of salt water has its fresh-water intake cut off, and then evaporates. These layers can occur more or less at random times in the geological history, and have characteristic fossils on either side.

Answer: These salt deposits are also referred to as “evaporites.” The problem here is that even the uniformitarian explanation above cannot account for the massive thickness of salt deposits if evaporation was the cause of it. Some salt deposits are so thick it would have taken the evaporation of a salt water body 130,000 meters high to form them.[108] So obviously there are other forces at work to cause these deposits other than evaporation.

The first thing that needs to be established is that the salt beds are more complex than just salt. They can contain gypsum, halite, magnesium and potassium salts.[109] The most notable deposits being that of halite (sodium chloride). Here we come across an issue because experiments conducted on salt water evaporation yield precipitated salts much different in composition[110] than what we observe in our salt beds (deposits) which is mostly halite. The only way water can precipitate halite while evaporating is if new sea water is constantly being added to a body of evaporating sea water maintaining the water volume while brine concentrations increase to the point that halite is precipitated.[111] As geologist Dr. Francis Pettijohn describes, “The crystallization of the brine is very complex, and depends not only on the solubility of the salts involved but also upon the concentration of the several salts present and the temperature… Inasmuch as many evaporate deposits show marked exceptions to the above requirements, simple evaporation of sea water did not occur, and either the parent brine was not formed from sea water or the evaporation took place under special conditions that will explain the anomalies.”[112]  Just what special conditions could have caused such deposits?

Dr. Snelling suggests that transportation of water is the key, “The ‘evaporite’ minerals have, in fact, been transported and reworked in the same way as the constituents of sandstones, siltstones, shales, and limestones. Transport can occur by normal fluid-flow processes, or by mass-transport processes such as slumps and turbidity currents. Turbidity currents transport mechanisms may have been particularly important in deposition of many ‘ancient’ deep-water evaporate deposits. Thus evaporate deposits display sedimentation  features the same as sandstones, siltstones, shales, and limestones, including both normal and reverse grain-size grading, cross-bedding and ripple marks. Such features are clearly related to water-transportand deposition.”[113]

Lastly, evaporation is slow and gradual and requires lots of time. But this would only be a problem if evaporation was the only method by which brine concentrated and precipitated salts. Thankfully, it is not. Volcanic waters and hydrothermal fluids are very rich with saline. When saline mixes with cold ocean water the sudden temperature drop causes the water mixture to become overly saturated with several salts. The water mixture cannot hold the salts, and thus precipitates it rapidly.[114] This is frequently observed around deep sea hydrothermal vents. Such actions were numerous at a large scale during the global flood, and can therefore account for the large salt beds we find today.

Problem 33: Hermatite Layers. Standard theory is that they were laid down before Earth’s atmosphere contained much oxygen. In an oxygen-rich regime, they would almost certainly be impossible.

Answer: This problem is assuming that there is only one way hermatite can form: ultraviolet rays hitting iron hydroxide forming hermatite.[115] This would require a level of ultraviolet rays so concentrated, they could have only penetrated earth’s atmosphere to reach the surface if there was no oxygen in the atmosphere.

But there is another way hermatite can form. The first clue is that hermatite is found in sedimentary rocks sandwiched between two volcanic layers, indicating they were formed in deep water anywhere from 200m to 1,000m. This is deduced from lack of erosion surfaces on the rocks, lack of wave or current textures, lack of subaerial volcanism, lack of bubbles in the surrounding volcanic rock, and the absence of aerosols, detrital material and volcanic ash.[116] Such formation of hermatite under water would require oxygen in the water, which would mean there was oxygen in the atmosphere. Furthermore, if the hermatite was formed by ultraviolet rays there would be crystals within the rock, but there is not.[117] So it can be concluded that the hermatite we find today was not formed in an oxygen-less environment, but instead in an oxygen-rich environment, and therefore does not conflict with the Flood model.

Problem 34: Organic Material. Where did all the organic material in the fossil record come from? There are 1.16 x 1013 metric tons of coal reserves, and at least 100 times that much unrecoverable organic matter in sediments. A typical forest, even if it covered the entire earth, would supply only 1.9 x 1013 metric tons.

Answer: First we should point out the “problems” with the uniformitarian model of how coal beds are formed. The uniformitarian model suggests that organic materials collected in peat swamps and condensed over long periods of time. Yet we find fauna and flora within these beds that could not have lived in a peat swamp environment. The peat swamp model also does not account for how widespread coal beds are across the continents. The coal beds are also interbedded with marine fossils… not likely in a peat swamp. The uniformitarian excuse is that the continents would form peat swamps, then sink below the ocean surface, then rise to form peat swamps again, then sink again, and so on and so forth. In that case, the IllinoisBasinwith its eighty marine layer sequences would have sank and risen below and above the ocean eighty times,[118] despite no geological system to facilitate such actions other than the pseudo-science claim that given enough time, anything is possible.

Instead what we should take note of is the field evidence of repeated deposited sediments and broken up floating vegetation within coal beds. Such evidence fits well with the flood model of floating debris being transported and buried by sediments where it in turn transformed to coal. The flood model would also explain well the interbedded marine fossils.

To address the claim that a global flood could not have collected enough vegetation to account for the large deposits of coal we see today requires a proper understanding of coal formation. Many skeptics like to inflate the amount of organic material needed to make coal, however studies show that eight to ten meters of compacted organic material can create one meter of coal,[119] or in some cases as little as five to six meters of organics can create one meter of coal.[120] Although this still wouldn’t account for all of today’s coal considering the current volume organic material on earth, it helps dramatically.

Next it should be addressed that the amount of organic material today was not necessarily the amount of organic material on earth pre-flood. There could have been much more. In fact, if continents were not as dispersed, and mountain ranges not as large pre-Flood as they are expected to have been, then there would have been large amounts of dense vegetation on what is now currently desert. Today desert covers one-fifth of the earth’s surface[121] with the ocean covering seven-tenths of the earth’s surface, it is easy to see how much of our land today is desert. This leads us to the other assumption that earth’s land surfaces are the same as they were during the flood. Yet, the pre-flood surface of earth was nothing like today’s earth. There could have indeed been much more land surface available for vegetation. Lastly, there is numerous fossil evidence of floating forest mats that once existed in shallow oceans.[122] Just how pervasive these floating forest mats where is unknown, but there abundance in the fossil record suggests they were very common. So in conclusion, in the pre-flood world with more land and more vegetation, there was ample organic material present to account for today’s coal beds.

Problem 35: Pervasive Aquatic Fossils. How do you explain the relative commonness of aquatic fossils? A flood would have washed over everything equally, so terrestrial organisms should be roughly as abundant as aquatic ones (or more abundant, since Creationists hypothesize greater land area before the Flood) in the fossil record. Yet shallow marine environments account for by far the most fossils.

Answer: This problem shows a lack of understanding of many basic natural observations, some of which is just common sense. Yes aquatic animals are more common in fossil records. But eighty percent of all the life in the world lives in the ocean.[123] So of course we would find more marine life in the fossil record than terrestrial life. In addition, terrestrial life was more capable of temporarily escaping the flood waters and being buried and subsequently fossilized. Marine life would not have had such ability. Terrestrial life is much more prone to floating than marine life. Floating would prevent burial and fossilization. And these are just basic factors at hand. When one considers the flood model and the uplifting of the sea floor depositing sediments onto land it is no wonder why marine fossils are much more abundant in the fossil record than terrestrial fossils.

POST-FLOOD

Problem 36: Plant Survival. How did modern plat species survive? A global flood would have surely killed off all plant life, so how did it come back?

Answer: Floating mats of vegetation are seen today and are even evident in the fossil record. Such floating mats would allow a whole host of plant life to survive. Seed bearing branches floating on the ocean surface would have sustained the seeds until the end of the flood. Even seeds themselves could have floated on the water until reaching dry land. These are the means by which it is believed vegetation made its way onto the Hawaiian Islands.[124] Seeds may have remained in the feathers and fur of animals aboard the ark. Seeds may have been stock piled for food, or even inadvertently in the food fed to the animals to be redeposited on land again once the ark landed on dry ground. There are a wide variety of possibilities for how plant life made it through the flood, all of which no doubt occurred to provide the variety of plants and trees available today.

Problem 37: Fish survival. How did fish survive the flood? Fish are very sensitive to water quality, salinity, temperature ect. Conditions during the flood would have killed all the fish!

Answer: There are some things that have to be clarified first. God’s Flood did not require that all aquatic animals survive the flood. It was only necessary for the smallest amount of a specific population, like fish, to survive in order to repopulate. But even then, many argue that there is nothing in Genesis that suggests God required any of the aquatic life to survive, His promise was only for that of land creatures (creatures of the “ground”) to survive. This would leave an opening for the extinction of many marine organisms during the flood. In any case, in the event of a global flood, turbulent, hot, and sediment filled waters with salinity and pH changes, the vast majority of sea life would perish. And evidently, as the fossil record shows, they did. But some suggest that the only necessary means, by which any sea life could survive at all, would require massive aquariums on board the ark for the sea creatures to live in. Which, of course, didn’t happen. So how did any sea life survive the flood?

Salinity: Though we cannot know what the salinity levels were in the ocean pre-Flood, we would expect the salinity to me more or less the same, and not drastically different as some suggest. Even as fresh water added to the oceans dropped salinity, salt precipitation from hydrothermal vents and erosion from the land would put salt back into the ocean. But of course this doesn’t solve the problem of sea life being very sensitive to salinity changes.

We should look to fish that can tolerate fresh and salt water environments. Fish like the goby, sawfish, salmon, striped bass, sea-run trout, Atlantic sturgeon and even types of eels live in both environments.[125] The key is to looking at fish families, in which case most fish families alive today have varieties that are both salt and fresh water fish, which is evidence of hybridization.[126] Catfish, salmon, herring, flatfish, ect., all have varieties that live in both fresh and salt water environments. This would suggest that the ability to adapt to salinity levels in the water was present in most fish prior to the flood. And in the time post-Flood, some fish became more specified to particular habitats while other retained their ability to adapt to both.

Temperature: Again, fish preference of temperature also may be a post-Flood specialization as families of fish are found in a wide variety of water temperatures. Today many fish can tolerate a wide variety of water temps if given sufficient time to adapt to them.[127]

Water Stratification: Often differences in water qualities don’t immediately equalize with each other, but instead can stratify for long periods of time. For example, waters with different salinities can rest on top of one another and not immediately mix. Fresh water running of the continents along with rain fall would have placed a fresh water blanket over many parts of the ocean. Fish washed from fresh water basins inland could survived in these fresh water layers.[128] There are observable conditions today in which both marine and fresh water organisms are found living in the same water column because of this stratification.[129] Thus the changes in salinity or temperature would have been survivable for fish during the flood. Stratification of water temperatures exist in bodies of water today as often colder more dense waters sink and warmer less dense waters rise.[130] The heat from tectonic activity from the ocean lithosphere would have surely warmed the cold waters along the bottom, definitely causing multiple layers of different water temperatures upon on another which certain fishes may have survived within.

So if fish had more capability to adapt and survive to changing water conditions and water stratification occurred, then there would be the possibility of, at the very least, a portion of the fish population to survive.

Problem 38: Disease Survival. How did diseases survive the flood?

Answer: This problem is loosely using the term disease. What exactly are we talking about? A disease is defined as a disturbance or anomaly in the functioning of the body that had a specific cause and identifiable symptom.[131] Disease can be caused by infection, poison, toxins, genetic mutations, etc. But surely this problem is referring to viruses and bacteria for the cause of disease so that is what we’ll look into.

If we are talking about disease, as in bacteria or viruses, than we must recognize that we are referencing their behavior as we observe today, in which many viruses and bacteria have specialized into having particular behaviors today that would have possibly made Flood survival impossible. This assumption doesn’t take into consideration that virus and bacteria behavior may have not been as differentiated as it is today. So a virus, for example, that requires a certain environment and host combination to survive not available during the flood, may not be an issue if that particular specialization had not occurred yet. It should also be understood that not all viruses and bacteria are bad. In fact some viruses and bacteria are essential to our survival,[132] like the bacteria in our digestive system.[133] Though many particular viruses and bacteria may have stemmed from these other harmless disease to form malicious ones or specialized one post-Flood. Such bacteria and viruses would have been carried within the animals aboard the ark, the organisms on the floating mats of vegetation, or in the surviving marine life, to spread in the post-Flood world.

Problem 39: Animal Dispersal. How and why did animals disperse so far from Mt Arat? How did they cross huge oceans? How could animals be so unique to certain regions. Shouldn’t animals of the same kind be found everywhere if they all dispersed from the same location. Shouldn’t there be fossils of all these animals be found nearMt.Arat? How could animals disperse all over the world so quickly?

Answer: The motivation to spread would of course be the same reason for animal dispersal today in which an increase in animal population requires the animals to spread outwards in search of new food sources and a home when the resources in their current environment are not enough to sustain them.[134] This would be a frequent occurrence in a landscape recovering from the flood in which vegetation was sparse. The animals that dispersed from theArk could have in fact traveled large distances through various means. Today we recognize that many animals (especially birds) are capable of traveling thousands of miles when migrating. But there are numerous other ways animals could have traveled over large distances or to lands separated by oceans They could have used land bridges, floating vegetation, or simply put, they could have swam.

Land Bridges: Land bridges today currently make it possible for animals to travel to almost every continent in the world. As biologist and zoologist Dr. Frank Marsh writes, “One glance at the world map will show that, with the exception of the narrow break at the bearing straight, a dry-land path leads from Armeniato all the lands of the globe except Australia. In the case of the latter the East Indieseven today form a fairly continuous bridge of stepping stones to that southern continent. As regards to the Bering Straight, there is no doubt that a land connection once existed between Asia and North America. With the straight closed, the cold waters of the Artic would have been prevented from coming south, and the Japan Current would have curved around the cast line farther north than today. The washing of those shores by warm waters of this current would have produced a dry-land route that even tropical forms could have used.”[135]

Floating Vegetation: As crazy as it sounds, floating vegetation or “islands” are observed to this day transporting animals and plant life large distances. Professor of Biology at the University of Vermont Paul Amos Moody writes of these islands: “In times of flood large masses of earth and entwining vegetation, including trees, may be torn loose from the banks of rivers and swept out to sea. Sometimes such masses are encountered floating in the ocean out of sight of land, still lush and green, with palms 20-30 feet tall. It is entirely probable that land animals may be transported long distances in this manner. Mayr records that many tropical ocean currents have a speed of at least 2 knots; this would amount to 50 miles in a day, 1000 miles in three weeks.”[136]

Paleontologist Dr. Alfred Sherwood Romer writes, “It seems certain that land animals do at times cross considerable bodies of water where land connections are utterly lacking… Floating masses of vegetation, such as are sometimes found off the mouths of the Amazon, may be one means of effecting this type of migration.”[137] The similarities between African and Madagascar animals and plants (a distance of 250 miles) has been chalked up to rafting.[138] In 1995, a hurricane in the Caribbean sent a matted “raft” of vegetation 30 ft wide carrying fifteen green iguanas over 190 miles away to the coast of Anguilla.[139] The green iguanas became established, andAnguilla now has a green iguana population.

Swimming: Whether voluntarily searching for new food sources, or involuntarily washed out to sea, there are numerous observed accounts of animals found far out at sea. Tigers and terrestrial snakes have been found over one kilometer out at sea. Elephants and cows have been seen crossing deep waters in search of food.[140]

The fossil dilemma, however, seems at first like an unavoidable problem. Take the Kangaroo for example; the kangaroo only lives in Australia, and the only fossils found of it are in Australia. There is no trail of fossils from where the kangaroo migrated. But this “dilemma” shows a lack of understanding of the fossilization process. Animal fossilization is a rare process, not a gradual process happening all the time. So we can’t use a lack of fossil evidence in a landscape as proof those animals were never there. For example, Lions used to roam through out the wilderness of Israeland surrounding regions until the Romans decimated the populations by capturing all of them for use in the arena displays. Yet, no Lion fossils have ever been found in Israeland the surrounding regions.[141] But we could not use this lack of fossil evidence to prove Lions never lived there, because we have numerous historical documentation that they did. Likewise, no fossils of kangaroos (or any other animal for that matter) in other regions does not mean they were never there.

There is also not a time issue as most people might think. Contrary to the assumption that animal migration takes thousands or millions of years, observable evidence shows just the opposite. Mt.Krakatoaerupted in 1883 causing massive destruction and devastation to all life on the island.[142] Yet 25 years later birds, insects, lizards, snakes, mollusks and earthworms were found on the island.[143] 50 years after the eruption, the barren island had become a dense tropical rainforest with 271 plant species and 31 species of bird, despite being 40 km away from the nearest land mass.[144] A similar event occurred in 1963 when the volcanic island of Surtsey off the coast of Iceland erupted killing all life on it. Within six months bacteria, mold, insects and birds were living on the island.[145] So even though these islands were separated from other land masses by large bodies of water, life had found its way back on the islands very quickly. With all this considered it is easy to see that the spreading of life post-flood is not a problem.

Problem 40: Inbreeding. All the animals would be inbreeding with each other to propagate the species. This would cause serious genetic problems.

Answer: Inbreeding would only be a problem if the particular animal populations remained small. However, in the post-flood world animal dispersal and repopulation would have been rapid, limiting any inbreeding to the first generation or so.[146] In addition, lethal recessive alleles accumulate with time, increasing the genetic load. Given the creation model time line of thousands of years, the animals that disembarked the ark would not have the accumulation of such alleles. Therefore, the brief inbreeding would not render genetic problems to the degree it would today within animal populations. Lastly, we can’t overlook the possibility of God choosing animal pairs with a very wide diversity of genes to avoid this problem as well.

Problem 41: Short lived species. How did short-lived species survive? Adult mayflies on the ark would have died in a few days, and the larvae of many mayflies require shallow fresh running water. Many other insects would face similar problems.

Answer: As said before, many of the attributes organisms have today are specializations which may have occurred post-Flood. Animals with short life spans could of course have reproduced, within the ark, or onboard the floating mats of vegetation. Mayflies may require such an environment today to reproduce, but such limitations may not have been necessary pre-Flood, as even the mayfly, just like the fish mentioned earlier, belongs to a family of flies which do not share this requirement for their larvae. This specialized trait may have developed post-Flood.

Problem 42: Devastated Habitat. How could more than a handful of species survive in a devastated habitat? The Flood would have destroyed the food and shelter which most species need to survive.

Answer: Again, as previously mentioned, such requirements for animal survival is assuming those animals have always required such, instead of recognizing the specialization of animal behavior in the new radically different environments found in the post-Flood world. Diets and shelters certain animals require today, may have not necessarily been the case pre-Flood, and instead developed in the post-Flood world. Ergo, it was the various different climates and lands in the post-Flood world that drove the differentiation of the animals to require such specialized diets, shelters, ect. In addition, as mentioned earlier in problem #39, animal and plant rebound and repopulation is very quick. Vegetation would be rapidly spreading in a nutrient rich barren landscape.

Problem 43: Carnivores. How could more than a handful of the predator species on the ark have survived, with only two individuals of their prey to eat? All of the predators at the top of the food pyramid require larger numbers of food animals beneath them on the pyramid, which in turn require large numbers of the animals they prey on, and so on, down to the primary producers (plants etc.) at the bottom. And if the predators survived, how did the other animals survive being preyed on?

Answer: This problem assumes that the only food available for carnivores in the post-Flood world were the other ark survivors. Yet, a post-Flood world would be littered with the rotting carcasses of all the creatures killed in the flood. Carnivores have a preference carrion over live animals, due to the massive amount of energy that needs to be exerted to hunt, chase and kill live animals.[147] Carrion is naturally the better option. The flood would have also left pools of water with trapped sea life. As that water evaporated the fish trapped in the pools would be easy pickings for carnivores on land. The bounty of easy sources of meat would have given herbivores sufficient time to repopulate to large enough numbers before the carnivores would have no choice but to turn back to hunting once the carrion was no more.

Problem 44: Population Rebound. How did the Human Population rebound so fast?

Answer: Human populations (with war, famine, disease, etc. taken into consideration) tend to double every 40 years.[148] Now let’s say you start with one man and one woman who reproduce and have offspring at this conservative rate. After 32 “doublings,” about 4,800 years, you’d have over 8 billion people.[149] Which is a lot more than there are currently in the world obviously. So if we go back to around 2,500 BC (the believed date of the global flood) and use a more conservative estimate of human populations doubling every 150 years, since sanitation and medicine was not as advanced to aid in population growth as it does today, then by today, 4,500 years later, you would have 6.5 billion people. So the population being what it is today, is not an issue for the Flood model. But it is however an issue for evolutionists. After all, if humans have been around for 50,000 years, the human population on earth should be 10100.[150] This is obviously not the case and needless to say, impossible.

Problem 45a: Historical Evidence. There should be historical records from various other cultures describing a global flood, not just Genesis.

Answer: There are! The ancient aboriginals have legends of a global flood that killed all life except for a handful of animals and people. The purpose of the flood being to wipe out all evil.[151] The Babylonians have an account of a global flood that was sent to kill all evil people, in which God ordered one man to build a boat for him, his family and the male and female of all the land animals to spare them from the flood.[152] The ancient Chinese have records of a flood that covered the mountains, in which a man built a large boat for his family and male and female of all animals to ride out the flood in.[153] In fact, there are over 80,000 flood accounts from ancient cultures worldwide.[154] Combining these accounts into regional areas and content, they compromise a total of 200 Flood traditions worldwide.[155] Of course this is circumstantial evidence, but evidence we would no doubt expect if the Flood actually happened. Such an important event would definitely be handed down through the various cultures and civilizations, which is why we find record of it in so many today.

Problem 45b: Differentiating Historical Evidence. Why do other flood myths vary so greatly from the Genesis account? Flood myths are fairly common worldwide, and if they came from a common source, we should expect similarities in most of them. Instead, the myths show great diversity. For example, people survive on high land or trees in the myths about as often as on boats or rafts, and no other flood myth includes a covenant not to destroy all life again.

Answer: Well now of course some skeptics are never satisfied and put the Genesis Flood account into a real “damned if you do, damned if you don’t” scenario. If Genesis was the only historical account of such a global flood, then it would not be counted as real evidence since no other ancient accounts support it. But Genesis is one of many global flood accounts, and yet, not satisfied, skeptics then nit pick all the details and assert that the global flood did not happen because all the stories don’t match exactly. This is why this problem has been broken down into A and B…

Yes, not all the stories are exact. Yet, isn’t it a bit of a coincidence that of all the traditions; 88% involved a favored family, 70% involved a boat to escape the flood, 95% claim man’s evil and needed extermination was the reason for the flood, and another 95% say the flood was global, all collaborating with the account in Genesis.[156] This is incredibly remarkable if one considers the limitations of communication and transportation in the ancient world. For such similarities worldwide to be a coincidence is highly unlikely unless there was a shared experience…

Theological scholar Allan MacRae writes, “If a universal flood occurred centuries after creation, it would be natural to expect that all humanity would recall many of it’s details for a long time, even though some points would tend to become quite garbled, as people more and more forgot the cause and purpose of the catastrophe.”[157]

Compared to all other narratives, the Genesis account is the longest, most detailed, and most compatible with that of eyewitness testimony. Many skeptics argue that many other flood accounts are older, the oldest being that of Native Americans some 8,000 to 10,000 years ago. And therefore the genesis writer(s) must have copied it. But these dates are taken from fallible carbon dating. In fact, fallible carbon dating aside, the oldest known accounts of the flood that can be determined by actual historical dates are that of the Genesis account and the Babylonian account. And most importantly, the Genesis writer(s) had no access to the traditions of all these other cultures if they were to copy them. But if the Flood account in Genesis is true, then naturally as humans dispersed post-Flood, the knowledge of the event would carry with them.

Problem 46: Egyptian History. The Egyptians (among others) have written records dating well back before 2250 B.C. (the Great Pyramid, for example dates to the 26th century B.C., 300 years before the Biblical date for the Flood). No sign in Egyptian inscriptions of this global flood around 2250 B.C.

Answer: This is what is known as Egyptian Chronology or Egyptology. The traditional chronology ofEgyptis incompatible with the Biblical record, and instead of investigating the Egyptology, skeptics instead assert the Bible is in error. However, if one were to research how Egyptology was established they would change their position on the matter.

In the 3rd century BC, Ptolomy II commissioned a priest named Manetho compile Egyptian history. Manetho compiled a list of pharaohs and the lengths of their reigns. He then applied them to particular calendar dates. This became known Manetho’s history and it is what Egyptology used as a foundation for its chronology.[158] But there are some significant problems with basing Egyptology off of Manetho’s history: Manetho’s history was never intended to be a chronological account of Egyptian history and it is inconsistent with contemporary Egyptian sources.[159]

Like other ancient historians, Manetho measured time in “regnal” years, not in a chronological time lime. A 4th century historian Eusebius, who used Manetho’s history, knew that Manetho’s regnal years were not to be added up consecutively, saying, “Several Egyptian Kings ruled at the same time… It was not a succession of kings occupying the throne one after another, but several kings reigning at the same time in different regions.”[160] Indeed many of the kings Manetho recorded were ruling simultaneously. Yet, when more modern historians began formulating the chronologies they placed the kings in sequential order, one after another. This is inaccurate and it stretched back Egyptian history further then it ever actually went.

Manetho’s history also doesn’t match up with contemporary Egyptian sources. Manetho often made judgment calls in his compilation that were uncalled for. For example, if a King was recorded with a different variation in the spelling of his name, Manetho assumed it was a different king altogether, thus creating several non-existing generations.[161] Professor of Egyptology at the University of Chicago, James Breasted writes of Manetho’s history as, “… a late, careless, and uncritical compilation, which can be proven wrong from the contemporary monuments in the vast majority of cases, where such documents have survived.”[162] Yet, the chronology of Egyptian dynasties and history was founded in Manetho’s historical account, and thus it is in error.

One way to authenticate an ancient civilization’s history is to match it up with other ancient historical accounts that reference similar events. In the many cases when Egyptian dates do not match up with other ancient accounts from another nation, sadly, the Egyptian account is considered the rule of thumb and the other nation’s dates are often considered in error… There are of course many other inconsistencies. Those who trust in carbon dating should be interested to know that carbon dating of artifacts often conflicts with the traditional Egyptology.[163] Historian Peter James writes of traditional Egyptology as a, “gigantic academic blunder.”[164] Archaeologist David Rohl writes, “The only real solution to the archaeological problems which have been created is to pull down the whole structure and start again, reconstructing from the foundations upward.”[165]

So in conclusion, it is not the Biblical record that should be challenged, but the traditional Egyptology that should be challenged. And therefore, Egyptian records spanning prior to the Flood is not a problem for the Flood account.

THEOLOGICAL AND PHILOSOPHICAL PROBLEMS?

Problem 47: Contradicting Text. How can a literal interpretation be appropriate if the text is self-contradictory? Genesis 6:20 and 7:14-15 say there were two of each kind of fowl and clean beasts, yet Genesis 7:2-3,5 says they came in sevens.

Answer: First we need to get a few things straight. Genesis 7:14-15 does not say the pairs were “clean,” but instead makes a general statement about pairs of every animal entering the ark. If one were to read the text they’d notice that Genesis 7:2-3 mentions that the seven pairs of animals brought on board were “clean animals,” while the others that just came in lone pairs were “unclean animals.” The difference being that clean animals were animals one could eat and sacrifice to God. The list of clean animals is a very narrow selection, and surely God allowed these animals on board for the family to eat. Hence, the animals came in pairs, the “clean” ones coming in seven pairs.

Now some skeptics suggest that the original text says they came in sevens, not seven pairs, thus contradict the other verses that the animals came in pairs because seven is obviously an odd number. As theologian John Upchurch explains, “’Seven’ is likely ‘seven pairs’ (the Hebrew allows for that), but even allowing that it means only ‘seven,’ it seems quite obvious that when describing the animals loading the Ark, the vast majority were pairs. God could have inspired Moses to say, ‘The animals loaded the Ark in pairs—with that seventh odd animal coming in behind as a slow tag-a-long.’ But not stating it that way doesn’t make it false; it’s just a simplification. If I write of a battle that ‘the troops came in two waves,’ that doesn’t preclude the possibility that the same troops attacked a first time and then a second. When describing a complicated event or concept, we often simplify things for brevity and understanding. Of course, if ‘seven’ is really ‘seven pairs’ (a good possibility), then the whole argument is moot anyway.”[166] Thus the text is not incontradiction.

Problem 48: Literal Cherry Picking. Why stop with the Flood story? If your style of Biblical interpretation makes you take the Flood literally, then shouldn’t you also believe in a flat and stationary earth? [Dan. 4:10-11, Matt. 4:8, 1 Chron. 16:30, Psalms 93:1, …]

Answer: No need to stop, because the Bible doesn’t say the earth is flat nor stationary:

-Daniel 4:10-11 is a dream or “vision,” as verse ten starts out by saying, of a tree so large it shaded all of earth. No where is it suggested that this dream reflects reality… and why would we assume a dream constituted reality anyways? We later read in Daniel that this dream was a prophecy of the end of King Nebuchadnezzar’s reign.

-Matthew 4:8 records Satan taking Jesus onto a high mountain to see all the Kingdoms of the World. How would it be possible to see all the Kingdoms of the world from a mountain unless the earth was flat? At this point though, we need to again take the verses in context. Satan appears to Jesus and instantly takes him to a high mountain top. Clearly this is a supernatural event! One must consider that this is also the third temptation of Satan to Jesus. The first takes place in the desert, the second on top of the temple, the third on top of the highest mountain. Clearly the anty is being upped as Satan is trying harder each and every time to tempt Jesus with power. Besides, even 1st century people were aware that even from a high mountain top you could not see the entire range of the earth (even if you thought it was flat).

-1 Chronicles 16:30 says the earth is firmly established and cannot be moved. See next;

-Psalms 93:1 says the world is established, firm and secure. Both words used for “establish” is kwun. Whereas the Hebrew word used for “move” is mowt.[167] Mowt is used elsewhere is Psalm 16:8 “I shall not be moved” (KJV). Does this mean the psalmist cannot move their body ever and is fixed in one place? Of course not. The psalmist can move, this verse is in context describing that he will not be swayed or lead astray from the path God has chosen for him. Likewise, in Psalm 93:1 and 1 Chronicles, the world will not be forced off its rotation and orbit because it is firmly established by God. But the real evidence that the Bible does not teach that the earth is flat comes from Job 26:10 and Isaiah 40:21-22 which indicate that the earth is “circular.”

The Greeks originally discovered the roundness of the Earth. Ancient civilizations were very knowledgeable of the earth being round considering the amount of attention they spent on observing the stars and other celestial objects in the night sky. Christian theologians by the numbers also disclaimed their belief in a round earth. In fact, only two Christian writers from the past have ever been recorded for promoting a flat earth, one a heretic from the 4th century named Lactantius and the other an Egyptian monk from the 6th century named Cosmas. So two… from the entire history of ancient Christian literature… There are many claims that Christians prosecuted and condemned those who thought the earth was round, but there is no historical evidence of this what so ever. Contrary to popular belief that the Church condemned Christopher Columbus from traveling West because they though he’d fall of the face of the planet, in reality the Church was well aware of the Earth being round. They condemnedColumbus because they believed the earth was too large forColumbus to circumnavigate cheaply. In other words, it was too expensive! The error the Church made was not the earth being flat, but instead thinking the ocean was too large.

In fact, no one has ever accused Christianity of being believers in a flat earth until the 19th and 20th centuries, when supporters of Darwinian Evolution were looking to discredit Christians challenging the theory. To do this they refer to Cosmas and Lactantius (ignoring the hundreds of other Christian authors distinctly not flat earthers) and the verses mentioned earlier, which only work when taken out of context. Clearly the Bible does not teach of a flat earth.

Problem 49: Parable Possibility. Jesus spoke in parables. So why would we think Genesis was not a parable also?

Answer: This is a logical fallacy of poor comparison. The Old Testament prophesized the messiah would teach in parables. The disciples referred to them as parables. Jesus acknowledged his teachings as parables. Jesus always explained what the parables meant. But the Genesis account is completely different. There is no mention by any figure that the Genesis narrative was a parable. In fact, Jesus took the Genesis account as actual history. This is also why no one explained the Flood story as to what it was suppose to represent (as the other parables were explained), because it was not a parable, but an actual historical narrative. There also is no separation in the structure of the text between parable and historical narrative. So if the Flood is a parable, then the entire book of Genesis would have to be a parable too. Even the parts that mention real historical figures and locations backed up by archaeology. But such figures and places are real and authenticated because the entire Genesis account is a highly detailed historical text. Not a generally vague parable like the ones Jesus used.

Problem 50: Omnipotent God. Why would an omnipotent God use a global flood to kill all of mankind. Why not kill them directly? And clearly this flood didn’t work because there is still evil in the world. So what was the point?

Answer: There really is no logical train of thought here. To ask why an omnipotent God did not kill in a particular way is in some way suggesting omniscience ourselves, as if to say, “If I was God I would have killed this way, not that way.” Why didn’t God kill the world with an asteroid? Why didn’t God hit everyone with lightening? Why didn’t God give them all heart attacks? There are countless ways God could have killed everyone, but His choice in “how” does not render the action invalid just because we think He was capable of doing it in a more efficient manner. To claim such would be suggesting you know better than an omniscient God. In the end, the why doesn’t matter if it actually happened the way it did.

Evil existed in the world. God used the Flood to cleanse the world of the evil. But eventually this evil would came back. The difference the second time around would be that God establishes a nation of His people. He establishes law with them and with a hope for redemption in the future with the Messiah, Jesus Christ, who eventually comes to earth as the ultimate sacrifice for our sins, and when returning, end sin, evil and pain for eternity. The flood is just one step in a long process to give everyone a chance to come to Him before all evil is ended.

We see this with God through out the Bible. There is separation, purification, judgment and redemption. This is seen with Noah, Jonah, Job, Paul, and ultimately everyone through Jesus. It is a testimony to an all just God, and an all loving God.

Peter sums it up best; Above all, you must understand that in the last days scoffers will come, scoffing and following their own evil desires. They will say, ‘Where is this ‘coming’ he promised? Ever since our ancestors died, everything goes on as it has since the beginning of creation.’ But they deliberately forget that long ago by God’s word the heavens came into being and the earth was formed out of water and by water. By these waters also the world of that time was deluged and destroyed. By the same word the present heavens and earth are reserved for fire, being kept for the day of judgment and destruction of the ungodly. But do not forget this one thing, dear friends: With the Lord a day is like a thousand years, and a thousand years are like a day. The Lord is not slow in keeping his promise, as some understand slowness. Instead he is patient with you, not wanting anyone to perish, but everyone to come to repentance.” -2 Peter 3:3-9 (NIV)

In conclusion, I hope I have provided with reason and logic, answers to the most profound claimed problems with the Biblical story of Noah’sArkand the Flood. It is my opinion that there is sufficient evidence, and where no evidence, sufficient reason, to believe that such an event is not only possible, but that it actually occurred. It may seem impossible to you regardless. But everything in the Bible is impossible. The story of the Exodus is impossible. Jesus walking on water, healing the sick, dying and returning from the dead is impossible. Of course such things would be impossible, but with an all powerful and all knowing God, such things ARE possible. Testimony to the glory and greatness of our sovereign God.


[1] Snelling, A., (2009) Earth’s Catastrophic Past, Vol. 1,Institute ofCreation Research:Dallas,TX, pp. 158.

[2] Ham, K., & Lovett, T., (2006) “Was There Really a Noah’s Ark & Flood?” as written in Ken Ham’s The New Answers Book 1, Master Books: Green Forest, AR, pp. 126.

[3] “CIVL 1101- History of Concrete,” Department of Civil Engineering,University ofMemphis; http://www.ce.memphis.edu/1101/notes/concrete/section_2_history.html

[4] Filby, F.A., (1970) The Flood Reconsidered: A Review of Evidences of Geology, Archaeology , Ancient Literature and the Bible, Pickering and Inglis Limited.:LondonEngland, pp. 93.

[5] Ramm, B., (1964) The Christian View of Science and Scripture, The Paternoster Press:Exeter, UK., pp. 157.

[6] Morris, H.M., (1971) “The Ark of Noah,” Creation Research Society Quarterly, 8(2): pp. 144.

[7] Hong, S.W., (1994) “Safety Investigation of Noah’s Ark in a Seaway,” Creation Ex Nihilo Technical Journal, 8(1): pp. 26-36.

[8] Lovett, T., (2010) “What Did Noah’s Ark Look Like?” as written in Ken Ham’s The New Answers Book 3, pp. 24.

[9] Snelling, A., (2009) Earth’s Catastrophic Past, Vol. 1,Institute ofCreation Research:Dallas,TX, pp. 133.

[10] Ham, K., & Lovett, T., (2006) “Was There Really a Noah’s Ark & Flood?” as written in Ken Ham’s The New Answers Book 1, Master Books: Green Forest, AR, pp. 129.

[11] Marsh, F.L., (1947) Evolution, Creation and Science, Review and Herald Publishing Association:WashingtonD.C., pp. 351

[12] Mayr, E., (1969) Principals of Systematic Zoology, McGraw-Hill Book Company:New York,NY, pp. 12.

[13] Jones, A.J., (1973) “Boundaries of the min: an analysis of the Mosaic lists of clean and unclean animals,” Creation Research Society Quarterly, 9(2): pp. 123.

[14] Woodmorappe, J., (2003) Noah’s Ark: A Feasibility Study, Institute for Creation Research:Santee,CA.

[15] Ham, K., & Lovett, T., (2006) “Was There Really a Noah’s Ark & Flood?” as written in Ken Ham’s The New Answers Book 1, Master Books: Green Forest, AR, pp. 129.

[16] Snelling, A., (2009) Earth’s Catastrophic Past, Vol. 1,Institute ofCreation Research:Dallas,TX, pp. 135.

[17] Woodmorappe, J., (2003) Noah’s Ark: A Feasibility Study, Institute for Creation Research:Santee, CA. pp. 16.

[18] Snelling, A., (2009) Earth’s Catastrophic Past, Vol. 1,Institute ofCreation Research:Dallas,TX, pp. 137.

[19] Woodmorappe, J., (2003) Noah’s Ark: A Feasibility Study, Institute for Creation Research:Santee, CA. pp. 21 and 98.

[20] Woodmorappe, J., (2010) “How Could Noah Fit the Animals on the Ark and Care for Them?” as written in Ken Ham’s The New Answers Book 3, Master Books:Green Forest,AR, pp. 53.

[21] The New Encyclopedia Britannica, (1985) Macropedia, Vol. 14, 670, article on “Dormancy.”

[22] Ransome, R., (1990) The Natural History of Hibernating Bats, Christopher Helm:LondonEngland, pp. 81.

[23] Snelling, A., (2009) Earth’s Catastrophic Past, Vol. 1,Institute ofCreation Research:Dallas,TX, pp. 152.

[24] Snelling, A., (2009) Earth’s Catastrophic Past, Vol. 2,Institute ofCreation Research:Dallas,TX, pp. 697-698.

[25] “Tsunamis: Killer Waves,” environment.nationalgeographic.com

[26] Lovett, T., (2010) “What Did Noah’s Ark Look Like?” as written in Ken Ham’s The New Answers Book 3, Master Books:Green Forest,AR, pp. 27.

[27] Snelling, A., (2009) Earth’s Catastrophic Past, Vol. 1,Institute ofCreation Research:Dallas,TX, pp. 145.

[28] Wieland, C., “The Lost Squadron,” creation.com

[29] Snelling, A., (2009) Earth’s Catastrophic Past, Vol. 2,Institute ofCreation Research:Dallas,TX, pp. 723.

[30] Snelling, A., (2009) Earth’s Catastrophic Past, Vol. 2,Institute ofCreation Research:Dallas,TX, pp. 695.

[31] Vail, P. R. & Mitchum, Jr. R. M., (1979) “Global Cycles of Relative Changes of Sea Level from Seismic Stratigraphy,” American Association of Petroleum Geologists Memoir 29, pp. 469-472.

[32] Snelling, A., (2009) Earth’s Catastrophic Past, Vol. 2,Institute ofCreation Research:Dallas,TX, pp. 699.

[33] Batten, D., “Tree Ring Dating (Dendrochronology),” creation.com

[34] Yamaguchi, D.K., (1986) “Interpretation of cross-correlation between tree ring series,” Tree Ring Bulletin, 46:47-54.

[35] Woodmorappe, J.,  (January 2009) “Biblical Chronology and the 8,000 Year-Long Bristlecone Pine Tree-Ring Chronology,” http://www.answersingenesis.org

[36] Batten, D., “Tree Ring Dating (Dendrochronology),” creation.com

[37] Snelling, A., (2009) Earth’s Catastrophic Past, Vol. 2,Institute ofCreation Research:Dallas,TX, pp. 703.

[38] Wise, K.P. & Austin, S.A., (1999) “Gigantic Megaclast within the Kingston Peak Formation (Upper Precambrian, Pahrump Group), South-Eastern California; Evidence for Basin Margin Collapse,” Geological Society of America Abstracts with Programs, 31: pp. 455.

[39] Snelling, A., (2009) Earth’s Catastrophic Past, Vol. 2,Institute ofCreation Research:Dallas,TX, pp. 705.

[40] Snelling, A., (2009) Earth’s Catastrophic Past, Vol. 2,Institute ofCreation Research:Dallas,TX, pp. 764.

[41] Perry, F.V., DePaolo, D.J. & Baldridge W.S., (1991) “Isotopic Evidence for a Decline in Crustal Contributions to Caldera-Forming Rhyolites of the Western United States During the Middle to Late Cenozoic,” Geological Society of America Abstracts with Programs, 23 (7): A441.

[42] Grande, L., (1984) Paleontology of the Green River Formation, with a Review of Fossil Fish Fauna, 2nd Ed., Geological Survey ofWyoming, Bulletin 63.

[43] Snelling, A., (2009) Earth’s Catastrophic Past, Vol. 2,Institute ofCreation Research:Dallas,TX, pp. 883.

[44] Hay, W.W., (1988) “Mass/Age Distribution and Composition of Sediments on the Ocean Floor and the Global Rate of Sediment Subduction,” Journal of Geophysical Research, 93 (B12): 14,933-940.

[45] Snelling, A., (2009) Earth’s Catastrophic Past, Vol. 2,Institute ofCreation Research:Dallas,TX, pp. 884.

[46] Snelling, A., (2009) Earth’s Catastrophic Past, Vol. 2,Institute ofCreation Research:Dallas,TX, pp. 881.

[47] Dott Jr., R.H. & Batten, R.L., (1988) Evolution of the Earth, 4th Ed., McGraw-Hill Book Company: New York, NY, pp. 155.

[48] Riddle, M. (2006) “Does Radiometric Dating Prove the Earth is Old?” as written in Ken Ham’s The New Answers Book 1, Master Books:Green Forest,AR, pp. 115.

[49] Snelling, A., (2009) Earth’s Catastrophic Past, Vol. 2,Institute ofCreation Research:Dallas,TX, pp. 800.

[50] Austin, S.A. (1996) “Excess Argon Within Mineral Concentrates from the New Dacite Lava Dome at Mount St Helens Volcano,” Creation Ex Nihilo Technical Journal 10(3): 335-343.

[51] Snelling, A. (1998) “The Cause of Anomlous Potassium-Argon ‘Ages’ for Recent Andesite Flows at Mt Ngauruhoe, New Zealand, and the Implications for Potassium-Argon ‘Dating,’ as written in R.E. Walsh’s Proceedings of the Fourth International Conference on Creationism, Creation Science Fellowship, Pittsburg, PA, pp. 503-525.

[52] DeYoung, D. (2005) Thousands… Not Billions, Master Books:Green Forest,AR, pp. 123-139.

[53] Riddle, M. (2006) “Does Radiometric Dating Prove the Earth is Old?” as written in Ken Ham’s The New Answers Book 1, Master Books:Green Forest,AR, pp. 120.

[54] DeYoung, D. (2005) Thousands… Not Billions, Master Books:Green Forest,AR, pp. 109-121.

[55] Snelling, A. (2008) “Doesn’t the Order of Fossils in the Rock Record Favor Long Ages?” as written in Ken Ham’s The New Answers Book 2, Master Books:Green Forest,AR, pp. 343.

[56] Morris, J.D. (1994) The Young Earth, Master Books:Green Forest,AR, pp. 70.

[57] Stanley, S.M. (2000) Earth and Life Through Time, 2nd Ed., Freeman and Company:New York,NY.

[58] Gignoux, M. (1955) Stratigraphic Geology, W.H. Freeman and Company:San Francisco,CA, pp. 46.

[59] Snelling, A. (2008) “Doesn’t the Order of Fossils in the Rock Record Favor Long Ages?” as written in Ken Ham’s The New Answers Book 2, Master Books:Green Forest,AR, pp. 349.

[60] Snelling, A. (2008) “Doesn’t the Order of Fossils in the Rock Record Favor Long Ages?” as written in Ken Ham’s The New Answers Book 2, Master Books:Green Forest,AR, pp. 349-350.

[61] “RapidCanyon Formation and Fossils,” Feedback Archive: February 19, 2011, Creation.com

[62] “RapidCanyon Formation and Fossils,” Feedback Archive: February 19, 2011, Creation.com

[63] Snelling, A. (2008) “Doesn’t the Order of Fossils in the Rock Record Favor Long Ages?” as written in Ken Ham’s The New Answers Book 2, Master Books:Green Forest,AR, pp. 350.

[64] Snelling, A. (2008) “Doesn’t the Order of Fossils in the Rock Record Favor Long Ages?” as written in Ken Ham’s The New Answers Book 2, Master Books:Green Forest,AR, pp. 350.

[65] Snelling, A., (2009) Earth’s Catastrophic Past, Vol. 2,Institute ofCreation Research:Dallas,TX, pp. 745.

[66] “RapidCanyon Formation and Fossils,” Feedback Archive: February 19, 2011, Creation.com

[67] Snelling, A., (2009) Earth’s Catastrophic Past, Vol. 2,Institute ofCreation Research:Dallas,TX, pp. 588.

[68] Van Andel, T.H. (1981) “Consider the Incompleteness of the Geological Record,” Nature, 294:398.

[69] Roth, A.A. (1988) “Those Gaps in the Sedimentary Layers,” Origins, 15:75-92.

[70] McKee, E.D. & Gutschick, R.G. (1969) “History of the Redwall Limestone in Northern Arizona,” Geological Society of America Memoir, 114: 202.

[71] Brand, L.R. (1979) “Field and Laboratory Studies on the Coconino Sandstone (Permian) Vertebrate Footprints and Their Paleoecological Implications,” Palaeogeography, Palaeoclimatology, Palaeoecology, 28:25-38.

[72] Snelling, A., (2009) Earth’s Catastrophic Past, Vol. 2,Institute ofCreation Research:Dallas,TX, pp. 504.

[73] Visher, G.S. (1990) Exploration Stratigraphy, 2nd Ed., Penn Well Publishing:Tulsa,OK, pp. 211-213.

[74] Snelling, A., (2009) Earth’s Catastrophic Past, Vol. 2,Institute ofCreation Research:Dallas,TX, pp. 505.

[75] Visher, G.S. (1990) Exploration Stratigraphy, 2nd Ed., Penn Well Publishing:Tulsa,OK, pp. 213.

[76] Allen, J.R.L. (1984) Sedimentary Structures: Their Character and Physical Basis, 2nd Ed., Elsevier Science Publisher:New York,NY, pp. 259-266.

[77] Snelling, A. (February 2008) “Catastrophic Granite Formation,” Answers Research Journal, http://www.answersingenesis.org.

[78] Petford, N. (1995) “Segregation of Tonalitic-Trondhjemitic Melts in the Continental Crust: The Mantle Connection,” Journal of Geophysical Research—Solid Earth 100B:15735–15743

[79] Harris, N., Vance, D. & Ayres, M. (2000) “From Sediment to Granite: Timescales of Anatexis in the Upper Crust,” Chemical Geology 162:155–167.

[80] Sawyer, E. W. (1991) “Disequilibrium Melting and Rate of Melt-Residuum Separation During Migmatization of Mafic Rocks from Grenville Front, Quebec.” Journal of Petrology 32:701–738.

[81] Snelling, A. (February 2008) “Catastrophic Granite Formation,” Answers Research Journal, http://www.answersingenesis.org.

[82] Petford, N.C., Kerr R.C. & Lister, J.R. (1993) “Dike Transport of Granitoid Magmas,” Geology, 21:845-848.

[83] Snelling, A. (February 2008) “Catastrophic Granite Formation,” Answers Research Journal, http://www.answersingenesis.org.

[84] Snelling, A. (February 2008) “Catastrophic Granite Formation,” Answers Research Journal, http://www.answersingenesis.org.

[85]  Pettijohn, F.J. (1957) Sedimentary Rocks, 2nd Ed., Harper & Row:New York,NY, Pp. 163.

[86] Lambert, A. & Hsu, K.J. (1979) “Nonannual Cycles of Varvelike Sedimentation in Walensee, Switzerland, Sedimentology, 26:453-461.

[87] Flint, R.F. (1971) Glacial and Quaternary Geology, John Wiley and Sons:New York,NY, pp. 406.

[88] Flint, R.F. (1957) Glacial and Pleisocene Geology, John Wiley and Sons:New York,NY, pp. 297.

[89] Flint, R.F. (1947) Glacial Geology and the Pleistocene Epoch, John Wiley and Sons:New York,NY, pp. 397.

[90] Ball, M.M., Shinn, E.A. & Stockman, K.W., (1967) “The Geological Effects of Hurricane Donna in South Florida,” Journal of Geology, 75: 583-597.

[91] McKee, E.D., Crosby, E.J. & Berryhill Jr., H.L., (1967) “Flood Deposits, Bijou Creek, Colorado, June 1965,” Journal of Sedimentary Petrology, 37: 829-851.

[92] Austin, S.A., (1986) “Mount St Helens and Catastrophism,” Proceedings of the First International of Creationism, Vol. 1, Creation Science Fellowship:Pittsburgh,PA, pp. 9.

[93] Buchheim, H.P. & Surdam R.C. (1977) “Fossil Catfish and the Depositional Environment of the Green River Formation, Wyoming,” Geology, 5:196-198.

[94] Snelling, A., (2009) Earth’s Catastrophic Past, Vol. 2,Institute ofCreation Research:Dallas,TX, pp. 954.

[95] Coffin, H.G. (1976) “Orientation of Trees in the Yellowstone Petrified Forests,” Journal of Paleontology, 50(3): 539-543.

[96] Snelling, A., (2009) Earth’s Catastrophic Past, Vol. 2,Institute ofCreation Research:Dallas,TX, pp. 955.

[97] Morris, J. (2010) “Do Fossils Show Signs of Rapid Burial?” as written in Ken Ham’s The New Answers Book 3, Master Books: Green Forest, AR, pp. 95-96.

[98] Coffin, H.G. (1987) “Sonar and Scuba Survey of a Submerged Allochthonous ‘Forest’ in Spirit Lake, Washington,” Palaois, 2:179-180.

[99] Sigleo, A.C. (1978) “Organic Geochemistry of Silicified Wood Petrified Forest National Park, Arizona,” Geochimica et Cosmochmica Acta, 42:1397-1405.

[100] Akahane, H., Furuno, T., Miyajima, H., Yoshikawa, T. & Yamamoto, S. (2004) “Rapid Wood Silicification in Hot Spring Water: An Explanation of Silicification of Wood During Earth’s History,” Sedimentary Geology, 169:219-228.

[101] McKee, E.D. & Gutschick, R.G. (1969) “History of the Redwall Limestone in Northern Arizona,” Geological Society of America Memoir, 114:103.

[102] Steinen, R. (1978) “On the Diagenesis of Lime Mud: Scanning Electron Microscopic Observations of Subsurface Material from Barbados, W.I.,” Journal of Sedimentary Petrology,  48:1139.

[103] Martill, D.M. (1989) “The Medusa Effect: Instantaneous Fossilization,” Geology Today, 5:201.

[104] Dill, R.F. & Steinen, R. (1988) “Deposition of Carbonate Mud Beds within High-Energy Subtidal Sand Dunes, Bahamas,” American Association of Petroleum Geologists Bulletin, 72:178-179.

[105] Snelling, A., (2009) Earth’s Catastrophic Past, Vol. 2,Institute ofCreation Research:Dallas,TX, pp. 926.

[106] Snelling, A., (2009) Earth’s Catastrophic Past, Vol. 2,Institute ofCreation Research:Dallas,TX, pp. 929.

[107] Wilson W.B. & Collier, A. (1955) “Preliminary Notes on the Culturing of Gymnodinium brevis Davis,” Science, 121:394-395.

[108] Snelling, A., (2009) Earth’s Catastrophic Past, Vol. 2,Institute ofCreation Research:Dallas,TX, pp. 938.

[109] Snelling, A., (2009) Earth’s Catastrophic Past, Vol. 2,Institute ofCreation Research:Dallas,TX, pp. 939.

[110] Clarke, F.W. (1924) The Data of Geochemistry, 5thEd.,US Geological Survey, Bulletin 770.

[111] Blatt, H., Middleton, G. & Murray, R., (1972) Origin of Sedimentary Rocks, Prentice-Hall:Englewood Cliffs, NJ, pp. 501-504.

[112] Pettijohn, F.J. (1957) Sedimentary Rocks, 2nd Ed., Harper and Rowe:New York,NY, pp. 483-484.

[113] Snelling, A., (2009) Earth’s Catastrophic Past, Vol. 2,Institute ofCreation Research:Dallas,TX, pp. 941.

[114] Hodes, M., Griffiths, P., Smith, K.A., Hurst, W.S., Bowers, W.J. & Sako, K. (2004) “Salt Solubility and Deposition in High Temperature and Pressure Aqueous Solutions,” American Institute of Chemical Engineers Journal, 50(9):2039-2049.

[115] Oard, M.J. (2010) “Did Early Earth’s Atmosphere Contain Oxygen?” Journal of Creation, 24(1):13

[116] Oard, M.J. (2010) “Did Early Earth’s Atmosphere Contain Oxygen?” Journal of Creation, 24(1):13-14

[117] Oard, M.J. (2010) “Did Early Earth’s Atmosphere Contain Oxygen?” Journal of Creation, 24(1):14

[118] Snelling, A., (2009) Earth’s Catastrophic Past, Vol. 2,Institute ofCreation Research:Dallas,TX, pp. 960.

[119] Diessel, C. (1992) Coal-Bearing Depositional Systems, Springer-Verlag:Berlin,Germany, pp. 12.

[120] Stach, E., Mackowski, M., Teichmuller, M., Taylor, G.H., Chandra, D. & Teichmuller, R. (1982) Stash’s Textbook of Coal Petrology, 3rd Ed., Gebruder Borntraeger: Berlin, Germany, pp. 17-18.

[121] “The Desert Biome,” University of California Museum of Paleontology, http://www.ucmp.berkeley.edu

[122] Snelling, A., (2009) Earth’s Catastrophic Past, Vol. 2,Institute ofCreation Research:Dallas,TX, pp. 962.

[123] “Ocean Facts,” marinebio.org

[124] “Evolution In Hawaii; A Supplement to Teaching About Evolution and the nature of Science,” http://www.nap.edu

[125] Snelling, A. (2010) “How Could Fish Survive the Genesis Flood?” in Ken Ham’s New Answers Book 3, Master Books:Green Forest,AR, pp. 198.

[126] Batten, D.J. (1999) “How Did Fresh- and Salt Water Fish Survive the Flood?” in Ken Ham’s The Answers Book: Updated and Expanded, Answers in Genesis:Brisbane,Australia, pp. 175-178.

[127] Snelling, A. (2010) “How Could Fish Survive the Genesis Flood?” in Ken Ham’s New Answers Book 3, Master Books:Green Forest,AR, pp. 200.

[128] Odum, E.P. (1971) Fundamentals of Ecology, W.B. Saunders:Philadelphia,PA, pp. 354.

[129] Snelling, A. (2010) “How Could Fish Survive the Genesis Flood?” in Ken Ham’s New Answers Book 3, Master Books:Green Forest,AR, pp. 201.

[130]  Raven, P.H. & Berg, L.R. (2004) Environment, 4th Ed., John Wiley & Sons, Inc.;Hoboken,NJ, Pp.119

[131] “Disease: What is a Disease?” http://www.ict-science-to-society.org

[132] Lightner, J.K. (2010) “Why Did God Make Viruses?” in Ken Ham’s New Answers Book 3, Master Books:Green Forest,AR, pp. 335.

[133] Gill, S. (2006) “Metagenomic Analysis of the Human Distal Gut Microbiome,” Science, 312:1355-1359.

[134] Marsh, F.L., (1947) Evolution, Creation and Science, Review and Herald Publishing Association:WashingtonD.C., pp. 291

[135] Marsh, F.L., (1947) Evolution, Creation and Science, Review and Herald Publishing Association:WashingtonD.C., pp. 291-292.

[136] Moody, P.A., (1953) Introduction to Evolution, Harper and Brothers:New York,NY, pp. 262.

[137] Romer, A.S., (1955) Vertebrate Paleontology, 2nd Ed.,University ofChicago Press;Chicago,IL, pp. 513.

[138] Tattersall, J., (1993) “Madagascar’s Lemurs,” Scientific American,  268(1) pp. 90-97.

[139] Censky, E.J., Hodge, K. & Dudley, J., (1998) “Over-water dispersal of lizards due to hurricanes,” Nature, 395(6702): 556.

[140] Brown J.H. & Gibson, A.C., (1983) Biogeography, The C.V. Mosby Co.:St. Louis,MO, pp. 314.

[141] Taylor, P., (2006) “How Did Animals Spread All Over the World from Where the Ark Landed?” as written in Ken Ham’s The New Answers Book 1, Master Books:Green Forest,AR, pp. 144.

[142] “How Volcanoes Work: KrakatauIndonesia (1883),” http://www.geology.sdsu.edu

[143] Carson, R.L., (1961) The Sea Around Us, OxfordUniversity Press:New York,NY, pp. 92.

[144] Brown, J.H. & Gibson A.C., (1983) Biogeography, The C.V. Mosby Co.:St. Louise,MO, pp.314

[145] “Community Ecology,” Encyclopedia Britannica, www.britannica.com

[146] Snelling, A., (2009) Earth’s Catastrophic Past, Vol. 1,Institute ofCreation Research:Dallas,TX, pp. 177.

[147] Woodmorappe, J. (2010) “How Could Noah Fit the Animals on the Ark and Care for Them?” in Ken Ham’s New Answers Book 3, Master Books: Green Forest, AR, pp. 56.

[148] U.S. Census Bureau, International Program, http://www.census.gov

[149] White, M. (September 2006) “Billions of People in Thousands of Years?” http://www.answersingenesis.org

[150] White, M. (September 2006) “Billions of People in Thousands of Years?” http://www.answersingenesis.org

[151] Coates, H. & Douglas W.H. (March 1981) “Ancient Aboriginal Flood Stories,” http://www.answersingenesis.org

[152] Lorey, F., “The Flood of Noah and the Flood of Gilgamesh,”Institute ofCreation Research, http://www.icr.org

[153] Traux, E.A., “Genesis According to the Miao People,”Institute ofCreation Research, http://www.icr.org

[154] Snelling, A., (2009) Earth’s Catastrophic Past, Vol. 1,Institute ofCreation Research:Dallas,TX, pp. 99.

[155] LaHaye, T.F. & Morris, J.D. (1976) The Ark on Arat, Thomas Nelson Inc.:Nashville,TN, pp. 233.

[156] Snelling, A., (2009) Earth’s Catastrophic Past, Vol. 1,Institute ofCreation Research:Dallas,TX, pp. 99.

[157] MacRae, A.A. (1950) “The Relation of Archaeology to the Bible,” Modern Science and Christian Faith, 2nd Ed., Van Kampen Press:Wheation,IL, pp. 234.

[158] Mitchell, E. (2008) “Doesn’t Egyptian Chronology Prove That the Bible Is Unreliable?” in Ken Ham’s New Answers Book 2, Master Books:Green Forest,AR, pp. 246.

[159] Mitchell, E. (2008) “Doesn’t Egyptian Chronology Prove That the Bible Is Unreliable?” in Ken Ham’s New Answers Book 2, Master Books:Green Forest,AR, pp. 246.

[160] As quoted in, Ashton, J. & Down, D. (2006) Unwrapping the Pharaohs, Master Books:Green Forest,AR, pp. 73.

[161] Mitchell, E. (2008) “Doesn’t Egyptian Chronology Prove That the Bible Is Unreliable?” in Ken Ham’s New Answers Book 2, Master Books:Green Forest,AR, pp. 247.

[162] As quoted in, Mackey, D. (1995) “Sothic Star Dating: The Sothic Star Theory of the Egyptian Calendar,” http://www.specialtyinterests.net

[163] Downs, D. “The Chronology of Egypt and Israel,” Diggings, http://www.biblicalstudies.qldwide.net.au

[164] James, P. (1991) Centuries of Darkness; A Challenge to the Conventional Chronology of Old World Archaeology, Jonathan Cape Ltd.: pp. 320.

[165] Rohl, D. (1995) Pharaohs and Kings: A Biblical Quest, Crown Publishers:New York,NY, pp. 9.

[166] UpChurch, J. (May 2008) “New Scientist Brings Up Old Arguments,” http://www.answersingenesis.org

[167] Faulkner, D., (August 2001) “Geocentrism and Creation,” Journal of Creation, 15(2): pp. 110.

Comments
  1. Pastafarianist says:

    Matthew, my man!!!

    Your explanations not worth a bean! Equally well I can prove existence of Santa Claus, Turtles Mutant Ninja and Spider-Man! Stop believe in stupid fairy tales about Flood and crucify Jewish Zombie. Flood it is an ordinary myth of ancient people, and Jesus is ordinary hippie jewish guy with masochist inclinations, who died many years ago.

    • matthew2262 says:

      I’ll take you up on that offer. Please prove to me the existence of Santa Claus, Turtles Mutant Ninja (which I believe are actually called the Mutant Ninja Turtles) and Spider-Man, “equally well,” as you put it.

      The examples you provide are a logical fallacy of poor comparison. The Ninja Turtles and Spider-man were purposely created with fiction in mind to entertain. Santa Claus is based off a combination of different traditions some based off real historical figures, some based in myth. Seeing how the Bible was created, maintained and consists of historical content in no way similar to these other examples is testimony to the poor comparison you’ve made.

      I also find it interesting that you would consider the flood myth ordinary among ancient people, when there are literally hundreds of different accounts that have shockingly a very similar story despite the lack contact between these ancient civilizations. That is a very unlikely coincidence to say the least, more so than just an ordinary happenstance.

      I will also reframe from commenting on your other opinions, since that is all they really are… opinions.

  2. […] See Matthew2262′s blog Share this:Like this:LikeBe the first to like this post. Tags: Ark, Flood, genesis, myth, Noah Permalink […]

  3. Not Many Wise says:

    I have found your thoughts interesting – there are a number of ideas I haven’t seen before.
    I have written a number of short posts about the Ark and the Flood, eg:
    http://notmanywise.wordpress.com/2011/12/16/some-facts-and-figures-about-noah%E2%80%B2s-ark/
    I’ve given your blog a mention in a new post.
    Thanks for your hard work.

  4. matthew2262 says:

    Heribert-Nilsson, Nils (former professor of botany, Lund University, Sweden, and member of the Royal Swedish Academy of Sciences) in the book Synthetische Artbildung p. 1194, describing the famous Baltic amber deposits:

    ‘The insects are of modern types and their geographical distribution can be ascertained. It is then quite astounding to find that they belong to all regions of the earth, not only the paleoarctic region as was to be expected. … The geological and paleobiological facts concerning the layers of amber are impossible to understand unless—the explanation is accepted that they are the result of an allochthonous [flood] process, including the whole earth.’

  5. matthew2262 says:

    Nicholas, writing in Scientific Monthly, May 1953 Vol 76, p.301, discussing the rich fossil deposits found in the Cumberland Bone Cave in Maryland. Flood geologists will have little difficulty accounting for this faunal mixing.

    ‘In this one cave there have been found such types as the wolverine, grizzly bear and Mustelidae, which are native to arctic regions. Peccaries, the most numerous type represented, tapirs and an antelope possibly related to the present-day eland are indigenous to tropical regions. Groundhogs, rabbits, coyotes and hare remains are indicative of dry prairies, but on the other hand such water-loving animals as beaver and muskrat suggest a more humid region.’

  6. matthew2262 says:

    But how good is the geological record? I have already mentioned that the ordinary viewpoint of evolution held by most paleontologists favours gradual incremental change. The fossil record, they say, is too incomplete to take seriously. And, they say, you cannot prove a gap.

    Illustration of gaps in the fossil record
    (Click image to enlarge)
    But of course you can prove a gap, especially if clines occurred. If there is a break in the record it must be possible to detect the break. The main point about breaks is that, if they were really random, as proposed by Darwin, they must have been plugged by one hundred and fifty years of work. But the gaps have not been plugged. They still persist; yet authorities still plead the cause of failure of preservation. Such authorities forget that if there is a million to one chance of one specimen of a population, and then if that species lived 5–15 m.y., we therefore get 5–15 times the population fossilized. The trouble may perhaps have lain more truthfully in our failure to find or describe the material. It is special pleading to rely upon gaps, and it is special pleading to propose inadequate preservation. We would do better to look at what the record really says.

    From the inagural lecture by J.B. Waterhouse.
    Assistant Prof. Geol. QLD University

  7. DD says:

    I am enjoying reading your defense but just a note to problem 1, Noah had 3 sons. (Gen 5:32)

  8. JA says:

    This is nice Matthew, well done!

  9. SM says:

    Hi, I stumbled across your blog while researching flood geology–I’m a former geologist and am thrilled to see a college student with the comprehensive knowledge and the media skill to get this information out there in a well-designed blog. Keep up the good work! Also, if you haven’t read it yet, Mike Oard’s 2011 book, “Dinosaur Challenges and Mysteries,” is an excellent summary of his Briefly Exposed Diluvial Surfaces hypothesis, which accounts for trace fossils such as footprints and ripple marks in a very plausible manner.

  10. Good to see that your blog is still here, so often we can quote a website only to find that later it has disappeared.
    Actually my own website, during a time of severe illness, I let my domain slip and have just now started with a new one!
    https://notmanywise.uk/
    I’ve just finished researching and updating an old post about ‘are dragons real creatures?’ obviously a lot of myth, but it’s been interesting that it must be based on some sort of dinosaur type creatures.
    Keep writing!

Leave a reply to SM Cancel reply